Ugc Net JRF June 2023 Shift 2 Master File

Download as pdf or txt
Download as pdf or txt
You are on page 1of 407

FREE

DISCLAIMER
This PDF is intended for educational purposes only. The content presented in
this PDF may include copyrighted material, the use of which has not been
specifically authorized by the copyright owner. We are making such material
available in an effort to advance understanding of various topics related to the
subject matter of this video. We believe this constitutes a 'fair use' of any
such copyrighted material as provided for in Section 107 of the Copyright Law.
If you wish to use copyrighted material from this video for purposes of your
own that go beyond 'fair use', you must obtain permission from the copyright
owner.
NTA NET JUNE 2023 (Shift-2)

ISSUES WITH NTA FINAL ANSWER KEY :-

Paper 1 Question ID – 902118 (Reading Comprehension Question)


Paper 2 – NO QUESTIONS DROPPED BY NTA.

Question ID – 902544 (IR Question on Principles of Just War)


Question ID – 902557 (IFP Question on India-US Strategic ties)
Question ID – 902560 (PPI Question on M.N. Srinivas)
NTA NET JUNE 2023 (Shift-2)

How this PDF will help you for DECEMBER Exam?

1. In-depth Analysis
2. Unit-wise solution + Detailed Explanation
3. Finding Sources of Questions
4. Unit-Wise Mind Map for DECEMBER 2023
5. Full Paper Section-wise Break-down
of Questions
NTA NET JUNE 2023 (Shift-2)

JUNE 2023 Questions : Unit wise Analysis

1. POLITICAL THEORY (9) 6. INDIAN FOREIGN POLICY (8)


2. WESTERN POLITICAL THOUGHT (9) 7. POLITICAL INSITUTIONS OF INDIA (8)

3. INDIAN POLITICAL THOUGHT (10) 8. POLITICAL PROCESSES IN INDIA (9)


4. COMPARATIVE POLITICS (9) 9. PUBLIC ADMINISTRATION (10)
5. INTERNATIONAL RELATIONS (13) 10. GOVERNANCE AND PUBLIC POLICY (5)
NTA NET JUNE 2023 (Shift-2)

JUNE 2023 Questions : Section wise Analysis

RED ICON MEANS =


BLUE ICON MEANS = GREEN ICON MEANS = Purple Icon Means= ORANGE ICON MEANS =
INDIRECT QUESTIONS
DIRECT FROM NTA DIRECT FROM NTA PYQs CURRENT AFFAIRS
THESE Qs. ARE OUTSIDE
PRESCRIBED SYLLABUS PRESCRIBED SYLLABUS SECTION SECTION
FROM NTA’s PRESCRIBED
(OLD REPEATED THINKERS) (NEW THINKERS 1ST TIME)
SYLLABUS
JUNE 2023 Shift – 2 Political Science Questions Section Wise Analysis

1%
11%

1% Out of 90 Political Science Questions –


DIRECT SYLLABUS - 35 Qs.
39%

NEW THINKERS/ NEW CONCEPTS - 43 Qs.

INDIRECT / OUTSIDE SYLLABUS - 1 Qs.

PYQs - 10 Qs.
48%
CURRENT AFFAIRS - 1 Qs.
POLITICAL
THEORY
DIRECT
NEW
CONCEPTS POLITICAL THEORY

1. Which among the following sentences are associated with the notion of equality
described by Amartya Sen?
A. The basic capability equality can be the sole guide to the moral good
B. Morality is not concerned only with equality
C. Basic Capability Equality is a partial guide to the part of moral goodness that is
associated with the idea of equality
D. The index of basic capabilities like utility, can be used in many different ways.
Choose the correct answer from the options given below:
1 B and D only 2 A and D only

3 A, B, C only 4 B, C, D only
DIRECT
NEW
CONCEPTS POLITICAL THEORY

1. Which among the following sentences are associated with the notion of equality
described by Amartya Sen?
A. The basic capability equality can be the sole guide to the moral good
B. Morality is not concerned only with equality
C. Basic Capability Equality is a partial guide to the part of moral goodness that is
associated with the idea of equality
D. The index of basic capabilities like utility, can be used in many different ways.
Choose the correct answer from the options given below:
1 B and D only 2 A and D only

3 A, B, C only 4 B, C, D only
EXPLANATION

▪ These lines are taken from Amartya Sen’s book, Choice, Welfare and Measurement (1982)
▪ He writes a chapter on ‘Equality of What?’ where he gives his concept of - ‘Basic Capability Equality’.

▪ Here, Amartya Sen tries to construct a Theory of Equality Rejecting the claims of Three Theories of Equality namely –
1) Utilitarian equality, 2) Total Utility equality, and 3) Rawlsian concept of equality.
▪ Amartya Sen's main thesis is none of these three is individually, nor is the combination of three a sufficient basis for
describing the Equality - Aspect of Morality.
▪ Amartya Sen argues that this gap can be narrowed by the idea of 'Basic Capability Equality’.

▪ The focus on basic capabilities can be defined - as a natural extension of John Rawls's concern with primary goods,
shifting attention from "goods" to "what goods do to human beings."
▪ Amartya Sen argues that Rawls's focus is limited to only the "goods" such as "income", rather than "on what income
does" to increase or decrease the capability of a person to access resources.
▪ If human beings were all very much alike each other, this would not have mattered a great deal, but there is evidence
says Sen, that the conversion of goods to capabilities varies from person to person substantially, and the equality of
former may vary a lot from the equality of latter.
EXPLANATION

▪ Statement A – “The basic capability, equality can be the sole guide to the moral good.” is False.
▪ In the book, Sen writes – “It is not my contention that the basic capability equality can be the sole guide to the moral
good.”

▪ Why Statement A is wrong?


▪ Amartya Sen argues that while Basic Capability Equality has clear advantages over other theories of equality –
but it cannot be considered as the sole guide to the moral good. He did not argue that the other theories
(Utilitarian equality, Total Utility equality, and Rawlsian concept of equality) are not morally irrelevant.
Therefore, the Basic Capability Equality is a partial guide to the moral goods that is associated with the idea of equality.

▪ Statements B, C, D are True and they are mentioned by Amartya Sen in his book.
▪ B. “Morality is not concerned only with equality.”
▪ C. “Basic capability equality is a partial guide to the part of moral goodness that is associated with the idea of equality.”
▪ D. “The index of basic capabilities like utility, can be used in many different ways.”
SOURCES

Choice, Welfare and Measurement By Amartya Sen (1982) Source : Choice, Welfare and Measurement By Amartya Sen by Pg. 369.
DIRECT
NTA
SYLLABUS POLITICAL THEORY

2. Which among the following rights is not group right, according to Will
Kymlicka?

1 Self-government rights
2 Polyethnic rights
3 Special representation rights
4 Right to speedy trial
DIRECT
NTA
SYLLABUS POLITICAL THEORY

2. Which among the following rights is not group right, according to Will
Kymlicka?

1 Self-government rights
2 Polyethnic rights
3 Special representation rights
4 Right to speedy trial
EXPLANATION
▪ Will Kymlicka argues that ‘Group Rights’ are necessary to protect the freedom and equality of individuals who belong to
minority groups.
▪ Without these Group rights, these individuals would be at risk of being assimilated into the majority culture, which could
lead to the loss of their cultural identity.
▪ Minority rights are ‘special’ rights, because they are specific to the groups to which they belong.
Each cultural group having different needs for recognition based on the particular character of its religion, traditions and
way of life.

▪ Kymlicka identified Three kinds of Minority Rights :-


▪ 1) Self-government Rights – Self-government rights apply to territorially concentrated national minorities, indigenous, or
tribal peoples, such as Native Americans in the USA; and First Nations, Inuits, and Metis in Canada.
It involves devolving political power to units controlled by members, possibly including the right of secession.
▪ 2) Polyethnic Rights – Polyethnic rights are rights that help ethnic groups and religious minorities, which have developed
through immigration, to express and maintain their cultural distinctiveness.
▪ 3) Special Representation Rights – Special representation rights aim to address underrepresentation of disadvantaged
groups in education and politics through "positive discrimination" to ensure equitable participation and policy
representation.
SOURCES

Source : Rapid Fire Revision Ebook (Eduseeker). Pg. 20.

Source : Political Ideologies by Andrew Heywood. Pg. 233.


DIRECT
(NEW
QUOTES) POLITICAL THEORY

3. Which among the following statements are associated with Harold J. Laski?
A. Every state is known by the rights that is maintains.
B. We do not mean by the rights the grant of some historic conditions possessed in
the childhood, but lost in the process of time.
C. Rights in fact are those conditions of social life without which no man can seek in
general to be himself at his best.
D. Rights are the legal opportunities for a successful life.
Choose the correct answer from the options given below :
1 A, B, C only 2 A, C, D only

3 A and C only 4 B and C only


DIRECT
(NEW
QUOTES) POLITICAL THEORY

3. Which among the following statements are associated with Harold J. Laski?
A. Every state is known by the rights that is maintains.
B. We do not mean by the rights the grant of some historic conditions possessed in
the childhood, but lost in the process of time.
C. Rights in fact are those conditions of social life without which no man can seek in
general to be himself at his best.
D. Rights are the legal opportunities for a successful life.
Choose the correct answer from the options given below :
1 A, B, C only 2 A, C, D only

3 A and C only 4 B and C only


EXPLANATION

▪ These lines are taken from Harold J. Laski’s book, A Grammar of Politics.

▪ Statements A, B, C are mentioned in the Chapter 3 - RIGHTS –


▪ A. Every state is known by the rights that is maintains.
▪ B. We do not mean by the rights the grant of some historic conditions possessed
in the childhood, but lost in the process of time.
▪ C. Rights in fact are those conditions of social life without which no man can seek
in general to be himself at his best.

▪ But, this statement – D. “Rights are the legal opportunities for a successful life.” is NOT given by Laski.
SOURCES

Source : A Grammar of Politics by Harold J. Laski. Chapter 3 – Rights.


DIRECT
NTA
SYLLABUS POLITICAL THEORY

4. Statement I: The socialist movement developed in France in the form of syndicalism.

Statement II : Syndicalism rejects the idea of class struggle.

1 Both Statement I and Statement II are true


2 Both Statement I and Statement II are false
3 Statement I is correct but Statement Il is false
4 Statement I is incorrect but Statement II is true
DIRECT
NTA
SYLLABUS POLITICAL THEORY

4. Statement I: The socialist movement developed in France in the form of syndicalism.

Statement II : Syndicalism rejects the idea of class struggle.

1 Both Statement I and Statement II are true


2 Both Statement I and Statement II are false
3 Statement I is correct but Statement Il is false
4 Statement I is incorrect but Statement II is true
EXPLANATION
▪ Only Statement I is True – “The socialist movement developed in France in the form of Syndicalism.” ✓
▪ Syndicalism is a type of radical trade unionism that advocates for the organization of workers into industrial unions (syndicates)
with the ultimate goal of achieving workers' control of the means of production.
▪ Syndicalism achieved great ideological success in France in the period between 1899 and 1937. Initially, the movement insisted
on the exclusive right of workers to control industry. But after the First World War (1914-18), it expanded its scope and
conceded the equal right of consumers in this sphere of control. Syndicalism stood for ‘Socialization Without State’.
▪ Syndicalism advocates the complete independence of Labour Unions from Political Parties.

▪ While Statement II is False – “Syndicalism rejects the idea of class struggle.” 


▪ Syndicalism argues the following points –
▪ 1) Syndicalism accepted the class-struggle theory of Marx.
▪ 2) Syndicalism preached abolition of the political state.
▪ 3) Syndicalism urged industrial action as the only effective means of bringing about a revolutionary change in society and
treated the 'general strike' as a means of securing workers' control over industry.
▪ 4) Syndicalism visualized a social order in which all power would be given to the producer; trade and industrial unions
would serve as the economic framework of society.
SOURCES

Source : An Introduction to Political Theory by O.P. Gauba. Pg. 46.


DIRECT
(NEW
THINKERS)
POLITICAL THEORY

5. Who among the following defined deliberative democracy in terms


of pluralistic association?

1 G. C. Sen 2 A Hanson

3 J. Cohen 4 P. Samuelson
DIRECT
(NEW
THINKERS)
POLITICAL THEORY

5. Who among the following defined deliberative democracy in terms


of pluralistic association?

1 G. C. Sen 2 A Hanson

3 J. Cohen 4 P. Samuelson
EXPLANATION

▪ Joshua Cohen, in his book chapter, “Deliberation and Democratic Legitimacy” –


defined Deliberative Democracy in terms of Pluralistic Association.
▪ Cohen defines Deliberative Democracy as an association whose affairs are governed by the
public deliberation of its members.
▪ Cohen gives Five main features on the Formal conception of Deliberative Democracy –
▪ 1) A deliberative democracy is an ongoing and independent association, whose members expect it to
continue into the indefinite future.
▪ 2) The members of the association share a commitment to coordinating their activities within institutions
that make deliberation possible
▪ 3) A Deliberative Democracy is a Pluralistic Association. The members have diverse preferences, convictions
and ideals concerning the conduct of their own lives.
▪ 4) Members of democratic associations value transparent deliberative procedures and prefer clear
connections between their deliberations and outcomes.
▪ 5) Members acknowledge each other's deliberative capacities, enabling them to engage in public reasoning
and act upon its outcomes.
SOURCES

Source : Joshua Cohen, 'Deliberation and Democratic Legitimacy’,


in R. Goodin and P. Pettit (eds.), Contemporary Political Philosophy:
An Anthology. Pg. 346.
DIRECT
(NEW
THINKERS)
POLITICAL THEORY

6. Who among the following scholars had levelled the criticism against
J.S. Mill that “we cannot separate two different compartments of
individual conduct”?
1 Barker 2 C. E. M Joad

3 L. T. Hobhouse 4 R. M. MacIver
DIRECT
(NEW
THINKERS)
POLITICAL THEORY

6. Who among the following scholars had levelled the criticism against
J.S. Mill that “we cannot separate two different compartments of
individual conduct”?
1 Barker 2 C. E. M Joad

3 L. T. Hobhouse 4 R. M. MacIver
EXPLANATION
▪ On the liberty of conduct, J.S. Mill draws a distinction between two types of actions of men: ‘Self-Regarding actions' and
‘Other-Regarding actions'.
▪ Mill advocates complete freedom of conduct for the individual in all matters not affecting the community, i.e. in the case
of ‘Self-Regarding actions'.
▪ However, in the case of ‘Other-Regarding actions', i.e. in matters which do affect the community, Mill argued for the
right of the community to coerce the individual if his conduct is prejudicial to its welfare.
▪ Thus, Mill defends complete freedom of conduct for the individual unless it adversely affects the community.
But the state could also interfere in the self-regarding action if it was thought to be very injurious to individual himself.

▪ Ernest Barker, in his book Principles of Social and Political Theory (1951), severely criticized Mill for separating the
conduct of individual into two parts—one which concerns others, and the other which merely concerns himself.
▪ Barker says, “We cannot separate two different compartments of individual conduct; but we can separate the sphere of
Society from that of the State. Because we cannot separate our individual conduct into two different compartments, and
because we are bound to regard the whole of our conduct as concerning others no less than ourselves, we have to admit
that the whole of our conduct is controllable—so far as the criterion of its concerning others is the criterion of judgment.”
▪ Barker suggested an alternative division of the conduct of individuals into the sphere of voluntary action managed by
society and regulated actions enforced by the state.
▪ Barker, in fact, has tried to shift Liberal theory from an Individualistic to a Pluralistic base.
SOURCES

Source : An Introduction to Political Theory by O.P. Gauba. Pg. 230.


FROM
NTA
PYQs POLITICAL THEORY

7. Chronologically arrange the following books in order of its publication year:

A. Liberty after Freedom


B. Four Essays on Liberty
C. The Constitution of Liberty
D. Two Concepts of Liberty

Choose the correct answer from the options given below:


1 A, B, C, D 2 B, C, D, A

3 C, A, B, D 4 D, C, B, A
FROM
NTA
PYQs POLITICAL THEORY

7. Chronologically arrange the following books in order of its publication year:

A. Liberty after Freedom


B. Four Essays on Liberty
C. The Constitution of Liberty
D. Two Concepts of Liberty

Choose the correct answer from the options given below:


1 A, B, C, D 2 B, C, D, A

3 C, A, B, D 4 D, C, B, A
EXPLANATION

▪ The Given Books Chronologically arranged in order of its publication year :-


➢ Two Concepts of Liberty – Isaiah Berlin (1958)
➢ The Constitution of Liberty – F.A. Hayek (1960)
➢ Four Essays on Liberty – Isaiah Berlin (1969)
➢ Liberty after Freedom : A History of Article 21, Due Process and the Constitution of India
– Rohan J. Alva (2022)
DIRECT
NTA
SYLLABUS
POLITICAL THEORY

8. Who among the following thinkers is an advocate of cosmopolitan


democracy?

1 Plato 2 Hegel

3 Kant 4 S. D. Lawrence
DIRECT
NTA
SYLLABUS
POLITICAL THEORY

8. Who among the following thinkers is an advocate of cosmopolitan


democracy?

1 Plato 2 Hegel

3 Kant 4 S. D. Lawrence
EXPLANATION

▪ At the core of Kant’s Cosmopolitan Democracy is the belief that the establishment of a World
Federation of Free Republics or Democracies would be the best means to achieve Perpetual Peace.

▪ Kant argued that Democracies, where power is vested in the hands of the People, are more inclined to
pursue Peaceful Relations with other nations. He believed that democracies, by their nature, would be less
likely to engage in aggressive wars and would prioritize diplomatic resolutions to conflicts.

▪ The three components of Kant’s hypothetical treaty for a Permanent Peace are –
▪ 1) First Definitive Article: The Civil Constitution of Every State shall be Republican.
▪ 2) Second Definitive Article: The Right of Nations shall be based on a Federation of Free States.
▪ 3) Third Definitive Article: Cosmopolitan Right shall be limited to Conditions of Universal Hospitality.

▪ NOTE :- Contemporary thinkers of Cosmopolitan Democracy are – Daniele Archibugi, David Held.
SOURCES

Source : Re-imagining Political Community: Studies in Cosmopolitan Democracy By Daniele Archibugi, David Held, Martin Köhler. Pg. 299,300.
TWO
NEW
CONCEPTS POLITICAL THEORY
AND THINKERS

9. Match List I with List II :


List-I (Books and Concepts) List-II (Thinkers)
A. Ethics of Care I. Kate Millet
B. Sexual Politics II. Carol Gilligan
C. Powers of Horror III. Julia Kristeva
D. Ecriture Feminine IV. Helene Cixous

1 A-I, B-II, C-III, D-IV 2 A-IV, B-III, C-II, D-I


3 A-II, B-I, C-III, D-IV 4 A-II, B-IV, C-I, D-III
TWO
NEW
CONCEPTS POLITICAL THEORY
AND THINKERS

9. Match List I with List II :


List-I (Books and Concepts) List-II (Thinkers)
A. Ethics of Care I. Kate Millet
B. Sexual Politics II. Carol Gilligan
C. Powers of Horror III. Julia Kristeva
D. Ecriture Feminine IV. Helene Cixous

1 A-I, B-II, C-III, D-IV 2 A-IV, B-III, C-II, D-I


3 A-II, B-I, C-III, D-IV 4 A-II, B-IV, C-I, D-III
EXPLANATION
▪ Ethics of Care – Carol Gilligan gave the concept of Ethics of Care as an alternative to
the hierarchal, principled approaches to ethics in her book In a Different Voice (1982).
▪ Gilligan, through her empirical research, argued against the empirical work
of moral psychologist Lawrence Kohlberg - who asserted that in general girls
and therefore women did not morally develop to the highest principled levels
while boys and men were more likely to develop a principled approach.
▪ Gilligan argued that Kohlberg's theory was gender-biased, as it privileged the voices of men over women.
▪ Gilligan presented an alternative empirical account that showed that girls and women approached ethical situations
differently than men, through connectedness and the primacy of caring relationships, instead of through abstract
principles that are the foundation for an ethics of justice.
▪ The Ethics of Justice is based on the idea that individuals have rights that must be respected. It emphasizes the
importance of fairness, impartiality, and following rules.
▪ Whereas, Gilligan’s Ethics of Care, on the other hand, is based on the idea that relationships are central to morality. It
emphasizes the importance of caring for others, being responsive to their needs, and building relationships based on
trust and mutual respect.
▪ The Ethics of Care has had a significant impact on feminist ethics and moral philosophy, providing a valuable perspective
that challenges the dominant theories while emphasizing the importance of empathy, compassion, and relationships in
moral decision-making.
EXPLANATION

▪ Sexual Politics – In the book Sexual Politics (1970), Kate Millett analysed the work
of male writers, from D. H. Lawrence to Norman Mailer, highlighting their use of
‘Sex’ to degrade and undermine Women.

▪ In her view, such literature reflects deeply patriarchal attitudes that pervade culture
and society at large, providing evidence that Patriarchy is a Historical and Social
Constant.

▪ Powers of Horror – The term “Abject" was introduced by French feminist


Julia Kristeva in her book "The Powers of Horror" (1980). It describes the strong fear
and disgust we feel when faced with aspects of ourselves and life that remind us of
our physical, messy nature. Bodily functions like waste and decay, as well as things
like corpses and maternal bodies, are examples of the abject according to Kristeva.

▪ Abject is an especially useful concept for feminists because Julia Kristeva


argues that All female bodies are viewed as inherently abject by Patriarchal Culture.
EXPLANATION
▪ Écriture Féminine– Écriture Féminine, or "feminine writing," is a concept developed
by French feminist theorist Hélène Cixous in her influential work "The Laugh of the Medusa,"
published in 1975. It is part of the broader feminist literary theory and aims to challenge
traditional male-dominated language and literature.

▪ The concept of Écriture Féminine advocates for a unique and distinct style of writing that arises from Women's
experiences, perspectives, and bodies. Cixous calls for women to reclaim their voices and identities by using language in
ways that break free from male-centered norms and conventions.

▪ According to Cixous, traditional Western literature has been predominantly written by men, with women often being
marginalized or portrayed through male perspectives. This creates a linguistic and cultural imbalance that suppresses
women's voices and perpetuates gender inequality.

▪ Écriture Féminine embraces fluidity, poetic expression, and ambiguity. It values emotions and intuition over strict logic
and reason, aiming to represent the complexity of women's experiences and perspectives.

▪ By embracing a more fluid and expressive language, Cixous believes women can reclaim their identities and challenge
oppressive gender norms.
SOURCES

Source : Feminist Philosophies A-Z by Nancy Arden McHugh. Pg. 19,39,69,165.


HOW TO PREPARE FOR DECEMBER EXAM 2023 ?

▪ To FOCUS ON –
▪ Read In-Depth, Read Pointwise
All the Arguments made by a Thinker
▪ Mark Keywords to Identify Unique aspect of Each Concept

▪ To FOCUS ON –
UNIT – 1 ▪ Political Ideologies – Read Background and Types of Each
Ideology. Focus on Info given in the Boxes of Textbooks.
POLITICAL THEORY

▪ Reference Material :-
▪ Introduction to Political Theory by O.P. Gauba
▪ Contemporary Political Philosophy by Will Kymlicka
▪ Political Ideologies by Heywood
▪ Eduseeker’s Ebook Rapid Fire Revision.
WESTERN
POLITICAL
THOUGHT
DIRECT
(NEW
QUOTES)
WESTERN POLITICAL THOUGHT

1. Who among the following thinkers hold the view that no citizen
should be sufficiently opulent to be able to purchase another and
none so poor as to be forced to sell himself?
1 Locke 2 Burke

3 Rousseau 4 Kant
DIRECT
(NEW
QUOTES)
WESTERN POLITICAL THOUGHT

1. Who among the following thinkers hold the view that no citizen
should be sufficiently opulent to be able to purchase another and
none so poor as to be forced to sell himself?
1 Locke 2 Burke

3 Rousseau 4 Kant
EXPLANATION
▪ Rousseau had written these lines in his major work The Social Contract –
▪ “Equality . . . must not be understood to mean that power and riches should be equally divided between all, but that power should never
be so strong as to be capable of acts of violence, or exercised but in virtue of the exerciser's station, and under direction of the laws, and
that in regard to riches, no citizen should be sufficiently opulent to be able to purchase another and none so poor as to be forced to sell
himself. This supposes on the side of the great, moderation in wealth and position, and, on the side of the lower classes, moderation in
avarice and greed.”
▪ Rousseau starts by dispelling the notion that equality implies an exact equal distribution of power and wealth.
▪ Absolute equality in these aspects is not what he advocates.
▪ Rousseau stresses the importance of limiting the concentration of power. He believes that no individual or group should hold such
overwhelming authority that they can use it to perpetrate violence or suppress the rights and freedoms of others. Instead, power should
be exercised responsibly and in accordance with General Will.
▪ Rousseau's concern here is with economic inequality. He argues against extreme wealth that could enable some individuals to have
control over others as if they were commodities to be bought and sold. Similarly, he opposes extreme poverty that would force people
into situations of desperation, leading them to sell their labor or services in degrading conditions.
▪ To achieve this vision of a just society, Rousseau calls for moderation on both sides.
▪ Therefore, for Rousseau it is not necessary that everyone be absolutely equal to everyone else, it is sufficient that no one single group
become so rich or so powerful that it can dominate others. Relative equality rather than absolute equality is Rousseau's goal.
SOURCES

Source : Western Political Thought – From Socrates to the Age of Ideology by Brian R. Nelson. Pg. 238.
DIRECT
NTA
SYLLABUS WESTERN POLITICAL THOUGHT

2. Which among the following thinkers hold the view that, liberty is destroyed
because the propertied classes possess the power to dominate others?
A. Rousseau
B. Marx
C. Locke
D. Engels

Choose the correct answer from the options given below:


1 B, C, D only 2 A, B, C only

3 A, B, D only 4 B and D only


DIRECT
NTA
SYLLABUS WESTERN POLITICAL THOUGHT

2. Which among the following thinkers hold the view that, liberty is destroyed
because the propertied classes possess the power to dominate others?
A. Rousseau
B. Marx
C. Locke
D. Engels

Choose the correct answer from the options given below:


1 B, C, D only 2 A, B, C only

3 A, B, D only 4 B and D only


EXPLANATION
▪ This question explores how the dominance of the propertied classes affects an individual's liberty.
▪ Thinkers who have argued this viewpoint are – Rousseau, Marx and Engels.
▪ Rousseau – In Second Discourse on the Origin and Foundations of Inequality, Rousseau traces the reason as
to why there has been a loss of freedom to express one's innate goodness in our society. This loss of freedom
is due to the inequality produced by the emergence of Private Property.
▪ Rousseau states that primitive people had no notion of private property, and therefore they all lived as
equals and enjoyed freedom as everyone else. With Civilization, came the emergence of different social
classes which gave rise to inequality of property.
▪ Rousseau says that along with social inequality there has also been a loss of liberty of individuals in both
political and personal sense. Hence, he argues that Political liberty is destroyed because the propertied
classes possess the power to dominate others.
▪ Marx and Engels – Marx and Engels claimed that liberty is undermined in capitalist societies because the
propertied classes (bourgeoisie) possess the economic power to dominate others (proletariat) and shape
societal institutions in a way that perpetuates their control and exploitation.
▪ Their solution was to advocate for the establishment of a classless society with collective ownership of the
means of production, where true individual freedom and liberty could be realized.
SOURCES

Source : Western Political Thought – From Socrates to the Age of Ideology by Brian R. Nelson. Pg. 224.
FROM
(ORIGINAL
TEXT) WESTERN POLITICAL THOUGHT

3. Gramsci holds the view that:


A. Progress is an ideology and becoming is a philosophical conception
B. Becoming is a philosophical concept from which progress can be absent
C. Progress depends on a specific mentality
D. Progress and becoming were born at the same time

Choose the correct answer from the options given below:

1 A, C, D only 2 A, B, C, D only

3 A and C only 4 B and D only


FROM
(ORIGINAL
TEXT) WESTERN POLITICAL THOUGHT

3. Gramsci holds the view that:


A. Progress is an ideology and becoming is a philosophical conception
B. Becoming is a philosophical concept from which progress can be absent
C. Progress depends on a specific mentality
D. Progress and becoming were born at the same time

Choose the correct answer from the options given below:

1 A, C, D only 2 A, B, C, D only

3 A and C only 4 B and D only


EXPLANATION
▪ These lines are taken from Antonio Gramsci's Prison Notebooks,
a collection of writings and reflections while he was imprisoned by the Italian Fascist regime.
▪ In this passage, Gramsci explores the distinction between “Progress" and “Becoming"
and their relationship to ideology and philosophical thought.
▪ "Progress is an ideology, Becoming is a philosophical conception."
▪ Gramsci starts by asserting that "progress" is an ideology, meaning it is a set of beliefs, values, and ideas that are shaped
by specific historical and cultural contexts. On the other hand, "becoming" is a philosophical concept, which suggests a
deeper, more fundamental idea about the nature of change and development.

▪ "Progress" depends on a specific mentality, in the constitution of which are involved certain historically determined
cultural elements: "becoming" is a philosophical concept from which "progress" can be absent.
▪ Gramsci explains that the idea of "progress" is tied to a particular mentality or worldview, shaped by historical and
cultural factors. It reflects a belief in moving forward, achieving improvement, and measuring development in
quantitative and qualitative terms.
▪ In contrast, "becoming" as a philosophical concept goes beyond these specific historical and cultural determinants. It
concerns the broader notion of transformation, change, and evolution, but without necessarily adhering to the idea of
linear progress or improvement.
EXPLANATION

▪ “Progress and becoming were born at the same time.”


▪ Gramsci points out that the ideas of "progress" and "becoming" came into prominence simultaneously.
▪ To illustrate the simultaneous emergence of these ideas, Gramsci draws an analogy to the birth of politics and philosophy.
Just as politics emerged in France and philosophy developed in Germany (later extending to Italy), the ideas of "progress"
and "becoming" also found their intellectual footing around the same time.

▪ In essence, Gramsci is distinguishing between “Progress" as an ideology with specific historical and cultural associations,
emphasizing notions of improvement and growth,
▪ and “Becoming" as a more abstract philosophical concept that involves change and transformation without necessarily
being tied to the idea of linear progress or quantifiable improvement.
SOURCES

Source : Selections from the Prison Notebooks of Antonio Gramsci. Pg. 358,359.
DIRECT
(NEW
QUOTES)
WESTERN POLITICAL THOUGHT

4. Who has stated that the idea of “praxis” is most clearly developed
in a short eleven paragraphs or thesis, that Marx had written to
himself in 1845?
1 H. J. Laski 2 B. R. Nelson

3 J. Wood 4 R. E. Lenser
DIRECT
(NEW
QUOTES)
WESTERN POLITICAL THOUGHT

4. Who has stated that the idea of “praxis” is most clearly developed
in a short eleven paragraphs or thesis, that Marx had written to
himself in 1845?
1 H. J. Laski 2 B. R. Nelson

3 J. Wood 4 R. E. Lenser
EXPLANATION

▪ B.R. Nelson in his book – Western Political Thought – From


Socrates to the Age of Ideology mentions that Karl Marx had
developed on the idea of “praxis” in a short eleven paragraphs or
thesis , that Marx had written to himself in 1845.

▪ These Eleven Theses were compiled into a book published in 1845


– ‘Theses on Feuerbach.’
SOURCES

Source : Western Political Thought – From Socrates to the Age of Ideology by Brian R. Nelson. Pg. 343.
FROM
NTA
PYQs
WESTERN POLITICAL THOUGHT

5. Which among the following thinkers can be equated with the


statement that, ‘He proposes an ascetic communication, the purpose
of which is to remove objects of desire’?
communism
1 Karl Marx 2 Plato

3 Glaucon 4 Polybius
FROM
NTA
PYQs
WESTERN POLITICAL THOUGHT

5. Which among the following thinkers can be equated with the


statement that, ‘He proposes an ascetic communication, the purpose
of which is to remove objects of desire’?
communism
1 Karl Marx 2 Plato

3 Glaucon 4 Polybius
EXPLANATION
▪ What is the meaning of Ascetic ?
▪ The term "ascetic" refers to a lifestyle or practice characterized by self-discipline and abstention from all
forms of indulgence, and simplicity. It is associated with staying away from all worldly pleasures,
material possessions, and physical comforts.

▪ Plato in his book ‘The Republic’ was deeply concerned with the concept of justice and believed that societal
harmony could only be achieved if individuals were in balance and devoid of excessive desires. He argued
that excessive material possessions and wealth could lead to greed, envy, and inequality, ultimately
disrupting the social fabric of the state.
▪ By removing the accumulation of wealth and possessions, Plato sought to create a society where everyone
focused on cultivating their virtues and contributing to the common good rather than pursuing individual
interests.
▪ It must be noted, that Plato’s communism applies only to the ruling elite, not to the vast majority of the
population. The artisans will be allowed to own private property and to have their families. The only purpose
of Plato's ascetic communism was to remove the objects of desire, not to distribute them more equitably.
SOURCES

Source : Western Political Thought – From Socrates to the Age of Ideology by Brian R. Nelson. Pg. 32.
DIRECT
NTA
SYLLABUS WESTERN POLITICAL THOUGHT

6. Match List I with List II :


List-I (Books) List-II (Authors)
A. On Violence I. Hannah Arendt
B. Reason and Revolution II. Herbert Marcuse
C. Where do we go from here III. Russell Kirk
D. The Conservative Mind : From IV. Martin Luther King Jr.
Burke to Elliot

1 A-I, B-II, C-IV, D-III 2 A-III, B-IV, C-II, D-I


3 A-I, B-II, C-III, D-IV 4 A-IV, B-III, C-I, D-II
DIRECT
NTA
SYLLABUS WESTERN POLITICAL THOUGHT

6. Match List I with List II :


List-I (Books) List-II (Authors)
A. On Violence I. Hannah Arendt
B. Reason and Revolution II. Herbert Marcuse
C. Where do we go from here III. Russell Kirk
D. The Conservative Mind : From IV. Martin Luther King Jr.
Burke to Elliot

1 A-I, B-II, C-IV, D-III 2 A-III, B-IV, C-II, D-I


3 A-I, B-II, C-III, D-IV 4 A-IV, B-III, C-I, D-II
EXPLANATION

▪ The Given Books matched with their respective Authors are as follows :-
➢ On Violence – Hannah Arendt
➢ Reason and Revolution – Herbert Marcuse
➢ Where Do We Go From Here – Martin Luther King Jr.
➢ The Conservative Mind : From Burke to Eliot – Russell Kirk
DIRECT
(NEW
QUOTES)
WESTERN POLITICAL THOUGHT

7. Who has stated that, if life only were the object, slaves and brute
animals might form a state?

1 Plato 2 Aristotle

3 Confucius 4 Hannah Arendt


DIRECT
(NEW
QUOTES)
WESTERN POLITICAL THOUGHT

7. Who has stated that, if life only were the object, slaves and brute
animals might form a state?

1 Plato 2 Aristotle

3 Confucius 4 Hannah Arendt


EXPLANATION
▪ Aristotle says, "A state exists for the sake of good life and not for the sake of life only, if life only were the object, slaves,
and brute animals might form a state."
▪ According to Aristotle, the State is the place where maximum conditions of free-speech exists.
▪ While, in the other Subordinate communities like the Household = free-speech is limited.

▪ For Aristotle, Politics is the highest expression of human's sociability and naturally the Polis or City-State is the space
where citizens can exhibit a greater degree of virtue (free-speech) than in their household or village.
▪ This is so because the Polis allows for the expression of reasoned action to a greater degree. This is why Aristotle argues
that slaves and animals cannot form a state. They lack free choice, therefore, cannot engage in any reasoned action.
▪ As a consequence, they cannot attain virtue. And since a virtuous life is the basis for a happy life - slaves or animals can
have no share in genuine human happiness.
▪ But, why does Aristotle assume that reasoned action can only occur in within the State? Why not within the household or
village? The reason is that in Subordinate communities - particularly the household, free choice is severely limited.
The function of the household is to maintain biological existence - eat, sleep and procreate.
▪ But, the State for Aristotle - is above mere biology. The State exists not only for the sake of life only, but it exists for the
arranging the conditions of good life. The State is a public arena in which people come together to make decision
affecting the whole community for good or evil. This requires, debate - in essence - free speech.
SOURCES

Source : Western Political Thought – From Socrates to the Age of Ideology by Brian R. Nelson. Pg. 57.
NEW TYPE
(BOOK’s
DETAILS Qs) WESTERN POLITICAL THOUGHT

8. Arrange the following chapters of Leviathan in correct sequence:


A. Of sense
B. Of imagination
C. Of reason and science
D. Of speech

Choose the correct answer from the options given below:

1 A, B, D, C 2 D, B, C, A

3 A, C, B, D 4 C, D, A, B
NEW TYPE
(BOOK’s
DETAILS Qs) WESTERN POLITICAL THOUGHT

8. Arrange the following chapters of Leviathan in correct sequence:


A. Of sense
B. Of imagination
C. Of reason and science
D. Of speech

Choose the correct answer from the options given below:

1 A, B, D, C 2 D, B, C, A

3 A, C, B, D 4 C, D, A, B
EXPLANATION
▪ Thomas Hobbes book Leviathan is written in Four Parts –
▪ Part I – Of Man
▪ Part II – Of Commonwealth
▪ Part III – Of a Christian Commonwealth
▪ Part IV – Of the Kingdom of Darkness
▪ Now, in Part I – the Correct Sequence of Chapters are –
▪ I: Of Sense
▪ II: Of Imagination
▪ III: Of the Consequence or Train of Imaginations
▪ IV: Of Speech
▪ V: Of Reason and Science
▪ How to Solve these type of Questions ?
▪ Along with Year-wise Chronological order of Books and Authors ✓
Do Prepare Notes On The Most Important Works ✓
See if some famous thinker has written the Preface, See the Chapters, See the Contents ✓
(Don’t do this for all books  . . . Only Cover The Most Important Books from Each Thinker ✓ )
SOURCES

Source : Leviathan by Thomas Hobbes. (1651)


NEW TYPE
(BOOK’s
DETAILS Qs) WESTERN POLITICAL THOUGHT

9. Frantz Fanon’s book ‘Black Skin, White Masks’ includes following chapters:
A. The Negro and language
B. The Negro and culture
C. The Negro and Psychopathology
D. The Negro and Recognition

Choose the correct answer from the options given below:

1 A, B, C only 2 B and D only

3 A and C only 4 A, C, D only


NEW TYPE
(BOOK’s
DETAILS Qs) WESTERN POLITICAL THOUGHT

9. Frantz Fanon’s book ‘Black Skin, White Masks’ includes following chapters:
A. The Negro and language
B. The Negro and culture
C. The Negro and Psychopathology
D. The Negro and Recognition

Choose the correct answer from the options given below:

1 A, B, C only 2 B and D only

3 A and C only 4 A, C, D only


EXPLANATION
▪ Frantz Fanon’s book Black Skin, White Masks has the following chapters –
▪ Foreword written by Homi Bhabha.
▪ Introduction
▪ Chapter 1 – The Negro and Language
▪ Chapter 2 – The Woman of Color and the White Man
▪ Chapter 3 – The Man of Color and the White Woman
▪ Chapter 4 – The So-Called Dependency Complex of Colonized Peoples
▪ Chapter 5 – The Fact of Blackness
▪ Chapter 6 – The Negro and Psychopathology
▪ Chapter 7 – The Negro and Recognition
▪ Chapter 8 – By Way of Conclusion
▪ How to Solve these type of Questions ? –
▪ Focus on what Keywords are associated with Frantz Fanon – Language, Psychopathology, Recognition, Race, Color,
Psychoanalysis, De-colonization, Mental Disease, Algiers Revolution.
▪ Now, Eliminate the words which are NOT associated with Fanon’s work – like Culture.
SOURCES

Source : Black Skin, White Masks by Frantz Fanon. (1952)


HOW TO PREPARE FOR DECEMBER EXAM 2023 ?

▪ To FOCUS ON –
▪ Cover Books and Authors – Year-wise Chronological Order
▪ Cover Major Books and their Themes, Contents, Imp. details
▪ Remember Keywords from Rapid Fire Revision ebook to use
in elimination method questions.

UNIT – 2
▪ To FOCUS ON –
▪ Prepare from One Standard Textbook for WPT
WESTERN ▪ Prepare from Rapid Fire Revision Guidebook for WPT
POLITICAL
THOUGHT
▪ Reference Material :-
▪ From Plato to Marx by Sushila Ramaswamy.
▪ Western Political Thought – From Socrates to the Age of
Ideology by Brian R. Nelson.
▪ Eduseeker’s Ebook Rapid Fire Revision.
INDIAN
POLITICAL
THOUGHT
DIRECT
(NEW
BOOK)
INDIAN POLITICAL THOUGHT

1. Who among the following is the author of the critical scholarly


memoir “Envoy to Nehru” ?

1 Escott Reid 2 Walter Crocker

3 Ramachandra Guha 4 Pratap Bhanu Mehta


DIRECT
(NEW
BOOK)
INDIAN POLITICAL THOUGHT

1. Who among the following is the author of the critical scholarly


memoir “Envoy to Nehru” ?

1 Escott Reid 2 Walter Crocker

3 Ramachandra Guha 4 Pratap Bhanu Mehta


EXPLANATION

▪ ‘Envoy to Nehru’ is a book by Escott Reid, a Canadian diplomat, who served as


High Commissioner to India from 1953 to 1957.

▪ The book Envoy to Nehru is a personal account of Reid's time in India, and it also
provides a detailed analysis of the special relationship that developed between
Canada and India during those years.

▪ Reid argues that the Canadian-Indian relationship was based on a common resistance
to the Cold War hysteria of the United States. He also credits the trust that developed
between Lester Pearson, the Canadian Prime Minister, and Jawaharlal Nehru,
the Indian Prime Minister, for the success of the two countries in working together on
issues such as the Korean War armistice (1953).
SOURCES

Source : Project MUSE. The Canadian Historical Review. Link : https://muse.jhu.edu/article/571013/pdf


DIRECT
NTA
SYLLABUS INDIAN POLITICAL THOUGHT

2. Which among the following statements is true about JP Narayan’s plan of


reconstruction of Indian polity?
A. He wanted to revive ‘swaraj from below’.
B. The basic unit of political organisation would be the Gram Panchayat.
C. The Gram Sabha would act as the basis of grass roots democracy.
D. This would also involves establishing an organic link amongst the different units
of the Panchayati Raj.
Choose the correct from the options given below:
1 A and B only 2 B and C only

3 C and D only 4 A, C and D only


DIRECT
NTA
SYLLABUS INDIAN POLITICAL THOUGHT

2. Which among the following statements is true about JP Narayan’s plan of


reconstruction of Indian polity?
A. He wanted to revive ‘swaraj from below’.
B. The basic unit of political organisation would be the Gram Panchayat.
C. The Gram Sabha would act as the basis of grass roots democracy.
D. This would also involves establishing an organic link amongst the different units
of the Panchayati Raj.
Choose the correct from the options given below:
1 A and B only 2 B and C only

3 C and D only 4 A, C and D only


EXPLANATION

▪ The context of the statements in question is related to JP Narayan's concept of Sarvodaya.

▪ Politically, Sarvodaya sought to establish a truly decentralized democracy that went beyond the democratic
elitism of the West and ensured what JP called Panchayati Raj or ‘Swaraj from below’.
▪ In Jayaprakash Narayan’s vision for Panchayati democracy under the idea of Sarvodaya, the way local
governance would work organically - is like this :
▪ 1. The basic unit is the Gram Sabha or Nagar Sabha, which is a gathering of all adults in a village, town, or
neighborhood.
▪ 2. From this Gram/Nagar Sabha, two different systems of government are formed at higher levels.
▪ 3. The first system is a three-tier local self-government. The Gram Sabha indirectly chooses the Panchayat
Samitis, which are assemblies representing a group of villages. And these Panchayat Samitis indirectly
choose the Zila Parishads, which are district-level assemblies.
▪ 4. The second system of legislative institutions that comes from the Gram/Nagar Sabha includes the Vidhan
Sabha (state assembly) and Lok Sabha (national parliament). These are elected through a three-step process.
SOURCES

Source : Indian Political Thought by M.P. Singh and Himanshu Roy . Pg. 266 .
DIRECT
NTA
SYLLABUS INDIAN POLITICAL THOUGHT

3. If we consider the Gandhian idea of Satyagraha as a dialogue, then which among


the following statement is not true?
A. The nature of Gandhi’s truth facilitates dialogues.
B. The nature of Gandhi’s truth is antithetical to equality.
C. In such a dialogue parties are constructed as the adversary.
D. The quest for truth is a shared exercise.
Choose the correct answer from the options given below:

1 A and B only 2 C and D only

3 B and C only 4 A and D only


DIRECT
NTA
SYLLABUS INDIAN POLITICAL THOUGHT

3. If we consider the Gandhian idea of Satyagraha as a dialogue, then which among


the following statement is not true?
A. The nature of Gandhi’s truth facilitates dialogues.
B. The nature of Gandhi’s truth is antithetical to equality.
C. In such a dialogue parties are constructed as the adversary.
D. The quest for truth is a shared exercise.
Choose the correct answer from the options given below:

1 A and B only 2 C and D only

3 B and C only 4 A and D only


EXPLANATION

▪ In the Gandhian idea of Satyagraha as a dialogue –

▪ The statements A and D are True.

▪ “The nature of Gandhi’s truth facilitates dialogues” – because Gandhi believed that truth is not something
that is possessed by one person or group. Truth is something that is shared by all people, and it can only be
found through dialogue and discussion. When people engage in dialogue, they are opening themselves up to
the possibility of learning from each other and of seeing the world from different perspectives. This can help
to break down misunderstandings and to build trust.

▪ “The quest for truth is a shared exercise” – Gandhi's philosophy emphasizes that the search for truth is a
collaborative effort. He believed that no one person or party possesses the ultimate truth, but it's through
the collective exploration of ideas and perspectives that a deeper understanding emerges. This shared quest
for truth brings people together and fosters a sense of unity and empathy.
EXPLANATION

▪ The statements B and C are False.


▪ “The nature of Gandhi’s truth is antithetical to equality” – Gandhi believed that all people are equal and that
they have the same right to truth. He also believed that everyone has the capacity to understand truth,
regardless of their social status or education level. When people engage in dialogue, they are acknowledging
each other's equality and their shared humanity.

▪ “In such a dialogue parties are constructed as the adversary” – This statement is false because Gandhi's idea
of dialogue and Satyagraha focused on finding common ground and understanding, rather than constructing
adversaries. He did not see the parties to a conflict as adversaries, but rather as fellow human beings who
are trying to find a solution to a problem. He believed that through open and respectful dialogue, even those
with differing viewpoints could come together to address conflicts and challenges.

▪ When people engage in dialogue, they are opening themselves up to the possibility of being changed by the
other person. They are also acknowledging that the other person has something valuable to contribute to
the conversation.
DIRECT
NTA
SYLLABUS
INDIAN POLITICAL THOUGHT

4. Who is the author of the book ‘Federation Versus Freedom’?

1 Pandita Ramabai 2 M.N. Roy

3 Dr. B.R. Ambedkar 4 Ram Manohar Lohia


DIRECT
NTA
SYLLABUS
INDIAN POLITICAL THOUGHT

4. Who is the author of the book ‘Federation Versus Freedom’?

1 Pandita Ramabai 2 M.N. Roy

3 Dr. B.R. Ambedkar 4 Ram Manohar Lohia


EXPLANATION
▪ The Book ‘Federation Versus Freedom’ was written by Dr. B.R. Ambedkar in the year 1939.

▪ In Poona the Gokhale Institute of Politics and Economics held an annual event to celebrate what they call the
Founder's Day. They invite someone to give a talk on a topic related to politics or economics.

▪ In 1939, Dr. Gadgil asked Dr. B.R. Ambedkar to deliver a speech. The invitation was accepted by
Dr. Ambedkar, and the subject he chose for the speech was – the Federal Scheme.

▪ The speech covered two main parts: (1) the Structure of the Federation and (2) a Critique of that Structure.

▪ On January 29, 1939, the speech was delivered at Gokhale Hall in Poona. Later, Dr. Ambedkar decided to publish the
entire speech and his notes as a book – ‘Federation Versus Freedom’.

▪ This book deals with the Following Chapters: Birth and Growth of the Federation, Structure of the Federation,
Powers of the Federation, Character of the Federation, Benefits of the Federation, The Bane of the Federation,
Fatality of the Federation, Federation without the States, Federation from different Points of View.
SOURCES

Source : Rapid Fire Revision ebook (Eduseeker). Pg . 70 .


DIRECT
NTA
SYLLABUS INDIAN POLITICAL THOUGHT

5. According to Kautilya, which among the following six methods of foreign policy is
applied by the would-be conqueror to the element of his Circle of States?
A. Making peace
B. Remaining non-violent
C. Staying quiet
D. Seeking support
Choose the correct answer from the options given below:

1 A, B, C, D only 2 A, B, C only

3 B, C, D only 4 A, C, D only
DIRECT
NTA
SYLLABUS INDIAN POLITICAL THOUGHT

5. According to Kautilya, which among the following six methods of foreign policy is
applied by the would-be conqueror to the element of his Circle of States?
A. Making peace
B. Remaining non-violent
C. Staying quiet
D. Seeking support
Choose the correct answer from the options given below:

1 A, B, C, D only 2 A, B, C only

3 B, C, D only 4 A, C, D only
EXPLANATION
▪ Option B is incorrect because Kautilya didn't advocate for a policy of remaining non-violent in his writings on foreign
policy. Instead, his approach to foreign policy was characterized by pragmatic realism and the use of power and strategy
to achieve the interests of the state.

▪ According to Kautilya’s Shadguna Niti, The Six Fold or Six Methods of Foreign Policy are as follows :-
▪ The King can adopt following options depending on situation
▪ 1. Sandhi (Treaty): If enemy is strong, go for sandhi. Ex: Shimla agreement (Pak-Ind).
▪ 2. Vighra: Break sandhi when you become strong.
▪ 3. Aasana (to sit): Stationing of forces near enemy's territory.
▪ 4. Yana (Movement): Military exercices near enemy territory.
▪ 5. Samashrya (Objective): Join hands with those who have similar objectives. Ex: Quad (to contain china).
▪ 6. Dvaidhbhava (Dual Policy): Friendship with one, enemity with other. Don't go for war at two fronts.
▪ Therefore, in the context of the given options, Kautilya's approach would align more with options A, C, and D.
Making peace (Option A) could be used strategically to consolidate power and resources, Staying quiet (Option C) might
be employed to gather information and avoid unnecessary conflict, and Seeking support (Option D) would involve
diplomatic maneuvers to form alliances and strengthen the state's position.
SOURCES

Source : Indian Political Thought ebook (Eduseeker). Pg . 19


.
DIRECT
(NEW
CONCEPTS)
INDIAN POLITICAL THOUGHT

6. According to Sri Aurobindo the task of “spiritual revolution” is


based on?

1 Truth consciousness 2 Parliamentary democracy

3 Radical humanism 4 Individualism


DIRECT
(NEW
CONCEPTS)
INDIAN POLITICAL THOUGHT

6. According to Sri Aurobindo the task of “spiritual revolution” is


based on?

1 Truth consciousness 2 Parliamentary democracy

3 Radical humanism 4 Individualism


EXPLANATION
▪ The revolution Sri Aurobindo pursued was a ‘Spiritual Revolution’.
▪ Spiritual revolution is that where the liberation of one opens the door to the liberation of all.
▪ The 'moral and spiritual' task of Aurobindo’s Spiritual Revolution is grounded upon three basic concepts of his philosophy -
Sachchidananda or the Supreme Reality, Truth Consciousness or the Supermind, and Evolution.
▪ Aurobindo describes The Supermind (or the Truth Consciousness) is the highest level of spiritual awareness. It's a state of consciousness
where you have all-knowing and all-powerful qualities that come from the divine.
▪ Think of it as a bridge connecting the highest spiritual reality (Absolute or Brahma) with regular human thinking (Mind). Aurobindo
called this bridge the 'Supermind.' This Supermind contains the true understanding of divine consciousness. Being in the state of
supermind means being fully aware of the Absolute, Bliss, and Brahma.
▪ Our regular thinking mind, which deals with ideas and knowledge, is limited compared to the supermind. The mind works with abstract
concepts, while the supermind has a more direct and spiritual understanding. However, according to Aurobindo, the real goal is to move
from the limited human mind towards the supermind for complete spiritual realization.
▪ Aurobindo believed that everything in reality is part of a spiritual evolution. This evolution is how consciousness grows and becomes
free. This doesn't mean that the ultimate divine consciousness is incomplete. Instead, the purpose of evolution is to allow the world and
humans to grow and develop spiritually. Evolution is like a gradual unfolding of the divine presence.
▪ There's a push from above, which encourages lower forms to evolve and reach higher levels of consciousness. At the same time, there's
an inner drive from below that helps this process. This evolution is purposeful because it's the way the Spirit expresses itself and shares
its essence with all levels of existence.
SOURCES

Source : Political Thought in Modern India by Thomas Pantham and Kenneth L Deutsch . Pg. 194.
DIRECT
(NEW
QUOTES)
INDIAN POLITICAL THOUGHT

7. Who said, “the truth is that the spirit of conflict and conquest is at
the origin and in the centre of the western nationalism’?

1 Swami Vivekanand 2 Rabindranath Tagore

3 M.K. Gandhi 4 Dr. B. R. Ambedkar


DIRECT
(NEW
QUOTES)
INDIAN POLITICAL THOUGHT

7. Who said, “the truth is that the spirit of conflict and conquest is at
the origin and in the centre of the western nationalism’?

1 Swami Vivekanand 2 Rabindranath Tagore

3 M.K. Gandhi 4 Dr. B. R. Ambedkar


EXPLANATION
▪ Rabindranath Tagore said, “The truth is that the spirit of conflict and conquest is at the origin and in the
center of the Western Nationalism.”
▪ Tagore writes this line in his book Nationalism, criticizing against the spirit of Western Nationalism which is
infused with aggressive nature which can be seen in the actions of the imperial powers of the West engaging
in conquest after conquest resulting into the first world war – World War I.
▪ Tagore says the nature of the Western state-system is flawed in itself. Tagore argues that the Western
concept of Nation-state has become a merely a political and economic union meant for a mechanical
purpose – which serves the state at the cost of the society.
▪ Previously, the state used to serve a specific purpose – it solely used to exist for the self-preservation of the
society.
▪ But, with time and the growing encroachments of the State – coupled with Industrialism, Consumerism –
gives birth to an unholy union – of the State and the Dominant Economic interests of the society.
▪ This results into the inevitable conquest and exploitation of other races. This idea of nation-state carries with
it an “exclusive mentality” which makes it easier for them to rationalize this aggressive behaviour.
▪ Therefore, this is why Tagore remarks that “the spirit of conflict and conquest is at the origin and in the
center of the Western Nationalism.”
SOURCES

Source : Political Thought in Modern India by Thomas Pantham and Kenneth L Deutsch . Pg. 183.
DIRECT
(NEW
QUOTES)
INDIAN POLITICAL THOUGHT

8. Who said, ‘Radicalism thinks in terms neither of nation nor class, its
concern is man; it concerns freedom as freedom of the individual’?

1 Sri Aurobindo 2 V. D. Savarkar

3 M. N. Roy 4 Deendayal Upadhyaya


DIRECT
(NEW
QUOTES)
INDIAN POLITICAL THOUGHT

8. Who said, ‘Radicalism thinks in terms neither of nation nor class, its
concern is man; it concerns freedom as freedom of the individual’?

1 Sri Aurobindo 2 V. D. Savarkar

3 M. N. Roy 4 Deendayal Upadhyaya


EXPLANATION
▪ M.N. Roy said, “Radicalism thinks in terms neither of nation nor class, its concern is man; it concerns
freedom as freedom of the individual.”

▪ As a Radical Humanist, Roy's approach was individualistic.

▪ He thought it's important not to place the individual under the control of a nation or a social group. Roy
rejected both the Congress party's focus on nationalism and the Communists' emphasis on class struggle.

▪ He said that Radical Humanism doesn't think in terms of nations or social classes; it's concerned with people
as individuals. According to Roy, freedom should be about the individual's freedom. He thought that
individuals shouldn't lose their uniqueness by getting absorbed into the identity of a nation or a social class.

▪ Roy didn't see much difference between a nation-controlled state and a class-controlled state. He believed
that both kinds of states limit individual freedom. He noted that modern democratic states still tend to
prioritize the collective identity of a nation or a class over the freedom of individuals.
SOURCES

Source : RADICAL HUMANISM OF M. N. ROY by B. K. Mahakul.


The Indian Journal of Political Science, Vol. 66, No. 3 (July-Sept.,
2005), pp. 607-618. Link : http://www.jstor.org/stable/41856152
DIRECT
NTA
SYLLABUS INDIAN POLITICAL THOUGHT

9. Arrange the writings of Dr. B. R. Ambedkar in a chronological order:


A. Annihilation of Caste
B. Who were the Shudras?
C. Future of Parliamentary Democracy
D. The Untouchables

Choose the correct answer from the options given below:

1 A, B, C & D 2 B, A, D & C

3 A, D, B & C 4 A, B, D & C
DIRECT
NTA
SYLLABUS INDIAN POLITICAL THOUGHT

9. Arrange the writings of Dr. B. R. Ambedkar in a chronological order:


A. Annihilation of Caste
B. Who were the Shudras?
C. Future of Parliamentary Democracy
D. The Untouchables

Choose the correct answer from the options given below:

1 A, B, C & D 2 B, A, D & C

3 A, D, B & C 4 A, B, D & C
EXPLANATION

▪ The writings of Dr. B. R. Ambedkar in a chronological order is as follows:

➢ Annihilation of Caste – 1936

➢ Who were the Shudras? – 1948

➢ The Untouchables – 1948

➢ Future of Parliamentary Democracy – 1951


SOURCES

Source : Rapid Fire Revision ebook (Eduseeker). Pg . 70 .


DIRECT
NTA
SYLLABUS INDIAN POLITICAL THOUGHT

10. Match List I with List II :


List-I (Thinkers) List-II (Concepts)

A. Rabindranath Tagore I. True freedom


B. J. P. Narayan II. Chaukhamba Model
C. Ram Manohar Lohia III. Hindu Rashtra
D. V. D. Savarkar IV. Sampurna Kranti

1 A-I, B-III, C-IV, D-II 2 A-III, B-IV, C-II, D-I


3 A-I, B-IV, C-II, D-III 4 A-IV, B-I, C-III, D-II
DIRECT
NTA
SYLLABUS INDIAN POLITICAL THOUGHT

10. Match List I with List II :


List-I (Thinkers) List-II (Concepts)

A. Rabindranath Tagore I. True freedom


B. J. P. Narayan II. Chaukhamba Model
C. Ram Manohar Lohia III. Hindu Rashtra
D. V. D. Savarkar IV. Sampurna Kranti

1 A-I, B-III, C-IV, D-II 2 A-III, B-IV, C-II, D-I


3 A-I, B-IV, C-II, D-III 4 A-IV, B-I, C-III, D-II
EXPLANATION
▪ The following Concepts and Their Thinkers matched are as follows :
▪ Rabindranath Tagore – True Freedom – There are four stages of true freedom according to Tagore –
(i) Realization of freedom at the individual level, (ii) at the community level, (iii) from community to universe,
and (iv) from universe to infinity.

▪ J.P. Narayan – Sampurna Kranti – The idea of Sampurna Kranti was first evolved by Vinoba Bhave during the 1960s and
this idea was later picked up by JP in 1975 against the emergency measures taken by the Indira Gandhi govt. JP did not
want to bring cosmetic reforms in the country. He wanted a Total Revolution in India – to transform the
socio-eco-political, cultural, spiritual and educational life of the people of India. Total Revolution would decentralize
all powers in the hands of the masses.

▪ Ram Manohar Lohia – Chaukhamba Model – Lohia’s concept of the four-pillar state is the manifestation of
decentralization of political and administrative power in the village, district, province and the center level.

▪ V.D. Savarkar – Hindu Rashtra – In Savarkar's perspective, a Hindu Rashtra would be a society and political system where
Hindu culture, traditions, and way of life would hold a central place. It emphasizes the cultural, religious, and historical
heritage of Hindus as the basis for shaping the nation's identity and ethos.
SOURCES

Source : Rapid Fire Revision ebook (Eduseeker). Pg . 61, 68 , 72 . -&- Indian Political Thought (Eduseeker) . Pg. 135 .
HOW TO PREPARE FOR DECEMBER EXAM 2023 ?

▪ To FOCUS ON –
▪ Read Timeline of Major Incidents, Events of IPT Thinkers
▪ Revise Books in Chronological Order from Revision ebook
▪ Stick to One Book. Get All the Concepts Clear. Get All The
Keywords Clear. Revise Books in Chronological Order Always.

UNIT – 3
▪ To FOCUS ON –
▪ Major Keywords Given By Each IPT Thinker
INDIAN ▪ Focus on the 2-3 Unique Keywords given by each Thinker
POLITICAL
THOUGHT
▪ Reference Material :- (Select Any One)
▪ Indian Political Thought by Himanshu Roy and MP Singh.
▪ Political Thought in Modern India by Pantham and Deutsch.
▪ Eduseeker’s Indian Political Thought (Detail) Ebook,
Eduseeker’s Ebook Rapid Fire Revision (IPT).
COMPARATIVE
POLITICS
DIRECT
NEW
CONCEPTS
COMPARATIVE POLITICS

1. “The Logic of collective action” explaining the mobilization of


interest groups in comparative political analysis is given by which of
the following scholars?
1 Theda Scokpol 2 Mancur Olson

3 Nicolas Poulatzas 4 Paul Pierson


DIRECT
NEW
CONCEPTS
COMPARATIVE POLITICS

1. “The Logic of collective action” explaining the mobilization of


interest groups in comparative political analysis is given by which of
the following scholars?
1 Theda Scokpol 2 Mancur Olson

3 Nicolas Poulatzas 4 Paul Pierson


EXPLANATION
▪ In 1965, American economist Mancur Olson challenged the Pluralist idea that group formation
was equally available to everybody.

▪ He believed that people's decisions are based on weighing the costs and benefits for themselves.

▪ In his 1965 book "The Logic of Collective Action" Olson argued that people are more likely to join groups that offer
specific rewards that they personally value (selective incentives).

▪ With the help of the concept of Rational choice, Olson suggests that only groups which offers individual benefits would
succeed.

▪ While groups offering benefits for the general public (public goods) would struggle to attract members, as these benefits
aren't tied to a person's membership.

▪ In other words, Olson suggested that people are more motivated to join groups that directly benefit them, rather than
those that offer benefits to everyone.
SOURCES

Source : Comparative Politics by Daniele Caramani . Pg. 259 .


DIRECT
NEW
CONCEPTS
COMPARATIVE POLITICS

2. Who amongst these is the proponent of the “Process Model” of


Social Movements in comparative politics?

1 Peter Evans 2 Charles Tilly

3 Doug McAdam 4 Ralph A Turner


DIRECT
NEW
CONCEPTS
COMPARATIVE POLITICS

2. Who amongst these is the proponent of the “Process Model” of


Social Movements in comparative politics?

1 Peter Evans 2 Charles Tilly

3 Doug McAdam 4 Ralph A Turner


EXPLANATION
▪ The context of this Question is – On Theories of Causes for the Emergence of Social Movements.
▪ The ‘Political Process Model’ of Movement Emergence was first given by Doug McAdam in 1982.
▪ Let us understand what is the ‘Political Process Model’ of Movement Emergence?
▪ The political process approach focuses on how social movements interact with their surroundings. This viewpoint also
helps us understand the various stages that social movements go through in its life cycle.
▪ This model has Two Main Arguments –
▪ Firstly, the political process approach argues that Social Movements don't just follow a set pattern internally, but are
also influenced by things outside like where does it stand on an issue, what resources they have to carry out the
movement, and how they strategize and deal with their rivals (movements’ opponents).
▪ Secondly, the political process model suggests that Social Movements happen when people who are organized, and are
aggrieved about something, and believe they can make a difference and take advantage of new chances in politics.
▪ According to Doug McAdam, John McCarthy, and Mayer Zald: a lot of political movements and revolutions start because
changes in society make the current political system weaker or more open to change.
▪ However, these "political opportunities" are just the first step. Without enough organization, whether formal or informal,
people won't likely take advantage of these chances.
▪ In conclusion, the 'Political Process Model' highlights how social movements interact with surroundings, influenced by
external factors and opportunities.
SOURCES

Source : Rethinking Social Movements : Structure, Meaning


and Emotion. Pg. 18 .

Source : Comparative Politics by Daniele Caramani . Pg. 284 .


INDIRECT
NOT
MENTIONED COMPARATIVE POLITICS
IN SYLLABUS

3. Arrange the following events of the U.S. constitutional development in ascending


order: Constitution
A. Establishment of the Federal Council of the U.S.A.
B. Introducing the Articles of Confederation of the U.S.A.
C. Addition of Bill of Rights in the U.S. Constitution.
D. 14th Amendment to the U.S. Constitution that guarantees equal protection of laws.

Choose the correct answer from the options given below:


1 A, C, B, D 2 C, A, D, B

3 B, D, C, A 4 B, A, C, D
INDIRECT
NOT
MENTIONED COMPARATIVE POLITICS
IN SYLLABUS

3. Arrange the following events of the U.S. constitutional development in ascending


order: Constitution
A. Establishment of the Federal Council of the U.S.A.
B. Introducing the Articles of Confederation of the U.S.A.
C. Addition of Bill of Rights in the U.S. Constitution.
D. 14th Amendment to the U.S. Constitution that guarantees equal protection of laws.

Choose the correct answer from the options given below:


1 A, C, B, D 2 C, A, D, B

3 B, D, C, A 4 B, A, C, D
EXPLANATION

▪ Events in the history of USA’s Constitutional Development in Ascending order:

➢ Introducing the Articles of Confederation of the U.S.A. – 1777

➢ Establishment of the Federal Constitution of the U.S.A. – 1787

➢ Addition of Bill of Rights in the U.S. Constitution – 1791

➢ 14th Amendment to the U.S. Constitution that guarantees equal


protection of laws – 1868
EXPLANATION
▪ In the history of the United States of America, these are some Landmark events in the development of their Constitution:

▪ Introducing the Articles of Confederation of the U.S.A. –


▪ The Articles of Confederation were adopted by the Continental Congress on November 15, 1777. This document
served as the United States' first constitution. It was in force from 1781 to 1789 when the present-day Constitution
went into effect.
▪ Establishment of the Federal Constitution of the U.S.A. –
▪ Written in 1787, ratified in 1788, and in operation since 1789, the United States Constitution is the world's longest
surviving written charter of government.
▪ Addition of Bill of Rights in the U.S. Constitution –
▪ On December 15, 1791, the new United States of America ratified the Bill of Rights, the first 10 amendments to the
U.S. Constitution, introducing Fundamental rights to the citizens of U.S.A.
▪ 14th Amendment to the U.S. Constitution that guarantees equal protection of laws –
▪ Passed by the Senate on June 8, 1866, and ratified two years later, on July 9, 1868, the Fourteenth Amendment
granted citizenship to all persons "born or naturalized in the United States," including formerly enslaved people, and
provided all citizens with “equal protection under the laws.”
SOURCES

Source : Comparative Politics by Daniele Caramani . Pg. 168 , 169 . Source : National Archives.
Home > Milestone Documents > Articles of Confederation (1777)
DIRECT
NTA
SYLLABUS COMPARATIVE POLITICS

4. Which of the following are the basic features of Eric Hobsbawm’s theory of nationalism
in Europe?
A. There are three phases of nationalism that have shaped states in Western countries.
B. He defines nationalism, for instance as reinventing traditions.
C. He identifies petty bourgeoisie as a group that advocated separatist nationalism.
D. He ignored the emergence of ethno-linguistic nationalism in East and West Europe.

Choose the correct answer from the options given below:


1 A, B, D only 2 B and D only

3 B and C only 4 A, C and D only


DIRECT
NTA
SYLLABUS COMPARATIVE POLITICS

4. Which of the following are the basic features of Eric Hobsbawm’s theory of nationalism
in Europe?
A. There are three phases of nationalism that have shaped states in Western countries.
B. He defines nationalism, for instance as reinventing traditions.
C. He identifies petty bourgeoisie as a group that advocated separatist nationalism.
D. He ignored the emergence of ethno-linguistic nationalism in East and West Europe.

Choose the correct answer from the options given below:


1 A, B, D only 2 B and D only

3 B and C only 4 A, C and D only


EXPLANATION
▪ Statements B and C are true –
▪ The Marxist historian, Eric Hobsbawm highlighted the extent to which nations are based on ‘Invented Traditions’.
Hobsbawm argued that a belief in the idea of a continuous history and pure culture is a myth and this myth
has been created by the idea of nationalism itself.

▪ The petty bourgeoisie were often the driving force behind separatist nationalism in Europe for a number of factors –
▪ 1. They were often excluded from political power. In many European countries, the merchant class was not allowed to
participate in government. This was because they were not considered to be part of the traditional ruling class. As a
result, they felt that they had no say in how their countries were run.
▪ 2. They were also often taxed very heavily. The merchant class was often taxed heavily by the government. This was
because they were seen as being wealthy. The high taxes made it difficult for merchants to do business and to grow their
businesses.
▪ 3. And they wanted to promote free trade for their own benefit. The merchant class wanted to be able to trade freely
with other countries. However, many European countries had restrictions on trade. These restrictions made it difficult for
merchants to do business and to grow their businesses.
▪ These were the reasons why Petty bourgeoisie became a driving force behind many of the Revolutions in the form of
Separatist Nationalism that took place in Europe in the 18th and 19th centuries.
DIRECT
NEW
THINKER COMPARATIVE POLITICS

5. Which of the following regimes of welfare states corresponds to


Richard Titmuss Institutional Redistribution Model in his three-fold
classification of Welfare State?

1 Anglo-Saxon Regime
2 Scandinavian social democratic regime
3 Conservative continental regime
4 Australian regime
DIRECT
NEW
THINKER COMPARATIVE POLITICS

5. Which of the following regimes of welfare states corresponds to


Richard Titmuss Institutional Redistribution Model in his three-fold
classification of Welfare State?

1 Anglo-Saxon Regime
2 Scandinavian social democratic regime
3 Conservative continental regime
4 Australian regime
EXPLANATION
▪ The context of this Question is on – Types of Welfare State Models given by Richard Titmuss –
▪ In the late 1950s, Richard Titmuss gave a three-fold model of Types of Welfare State regimes as follows :

Welfare State
Regimes

Residual Welfare Model – Industrial Achievement Performance Institutional Redistributive Model –


Model – In this model of welfare state Social
In this kind of welfare system, help
is offered only when the usual ways In this type of welfare system, the Welfare Institutions are an integral
like private market and family can't benefits you get are connected to part of society, providing Universalist
meet people's social needs anymore. your job, and how well you work. It's Services to everyone, not just
based on your effort and how through buying and selling.
productive you are. Examples – Sweden, Norway,
Examples – United States, Examples – Germany, Japan, Denmark. (Scandinavian
United Kingdom, Canada. South Korea. Social - Democratic Regimes).
SOURCES

Source : Comparative Politics by Daniele Caramani .Pg. 384,385.


DIRECT
NEW
THINKER COMPARATIVE POLITICS

6. Which of the following statements given by Juan Linz and Alfred Stepan are true?
A. Both civil society and political society are complementary to each other.
B. The complimentary is always recognized.
C. Rule of law is condition for consolidation of democracy.
D. Modern consolidated democracies have pure market economies only.
E. Modern democracies must have effective capacity to command, regulate and extract.

Choose the most appropriate answer from the options given below:
1 A, C and E only 2 A and B only

3 B, C and E only 4 B, C and D only


DIRECT
NEW
THINKER COMPARATIVE POLITICS

6. Which of the following statements given by Juan Linz and Alfred Stepan are true?
A. Both civil society and political society are complementary to each other.
B. The complimentary is always recognized.
C. Rule of law is condition for consolidation of democracy.
D. Modern consolidated democracies have pure market economies only.
E. Modern democracies must have effective capacity to command, regulate and extract.

Choose the most appropriate answer from the options given below:
1 A, C and E only 2 A and B only

3 B, C and E only 4 B, C and D only


EXPLANATION
▪ The context of this Question is – On Democratic Transition and Consolidation.
▪ These statements given by Juan Linz and Alfred Stepan are taken from their book – ‘Problems of Democratic Transition
and Consolidation’ where they write on Democratic Transition and Consolidation in three regions - Southern Europe,
South America, and Post-Communist Europe.
▪ Out of these Linz and Stepan argue that in Southern Europe we see Completed Consolidations, in South America we see
Constrained Transitions and in Post-Communist Europe we see most complex paths towards democratic transition.
▪ Statements A, C and E are true –
▪ A. Both civil society and political society are complementary to each other.
▪ C. Rule of law is condition for consolidation of democracy.
▪ E. Modern democracies must have effective capacity to command, regulate and extract.
▪ Statements B and D are false –
▪ B. The complimentary is always recognized. Why? – Within the democratic community, champions of either Civil or
Political Society adopt practices which limits the development of the other. They remain in opposition.
▪ D. Modern consolidated democracies have pure market economies only. Why? – To form supportive conditions for a
Consolidated democracy we need neither command economy or pure market economy, - instead – what we need is
“economic society” – one which mediates between the state and market.
SOURCES

Source : Problems of Democratic Transition and Consolidation by Juan Linz and Alfred Stepan . Pg. 8,9,10,11 .
DIRECT
NTA
SYLLABUS COMPARATIVE POLITICS

7. Assertion (A): There is a significant ideological difference between political


parties in a polarized multiparty system with a strong dose of dogmatic
radicalism.

Reason (R): Political coalitions in polarized multiparty system are viable.

1 Both A and R are true and R is the correct explanation of A


2 Both A and R are true but R is NOT the correct explanation of A
3 A is correct but Statement R is false
4 A is not correct but Statement R is true
DIRECT
NTA
SYLLABUS COMPARATIVE POLITICS

7. Assertion (A): There is a significant ideological difference between political


parties in a polarized multiparty system with a strong dose of dogmatic
radicalism.

Reason (R): Political coalitions in polarized multiparty system are viable.

1 Both A and R are true and R is the correct explanation of A


2 Both A and R are true but R is NOT the correct explanation of A
3 A is correct but Statement R is false
4 A is not correct but Statement R is true
EXPLANATION
▪ The context of this Question is – Multiparty System and degree of their polarization
(Sharp division of beliefs, leading to extreme differences in party politics.).
▪ Giovanni Sartori argues that the way in which multiparty systems function largely depends on the degree to which parties
are ideologically polarized. He has distinguished two main types of multiparty systems : Moderate Multiparty systems
and Polarized Multiparty systems.
▪ In Moderate Multiparty Systems – the number of parties is small and the direction of the competition is Centripetal, i.e.
the main parties tend to converge towards the centre of the left–right scale to attract the support of the moderate
electorate. Since here the differences between parties are small, coalitions can form easily.
▪ While in Polarized Multiparty Systems – “There is a large ideological distance between parties with a strong dose of
radicalism.” There is a one main party placed at the centre of the left–right axis which represents the ‘system’ against
which extreme anti-system parties are opposed. These Anti-system parties aim to change not only government but also
the system of government.
▪ Thus, “not all coalitions are viable,” with some parties continuously excluded from coalitions and remain in constant
opposition. They become irresponsible and radicalize with promises they will never be called to put into practice.
When one party controls the center, it stops other parties from moving towards it. This leads to divergence and
competition is Centrifugal (away from center).
▪ Hence, as per the Question – Statement A is true as quoted above, and Statement B is false as Political Coalitions in
Multiparty Polarized systems are NOT viable in the long run.
EXPLANATION

Political Spectrum of Parties

Moderate Multiparty System Polarized Multiparty System

▪ Ideological Differences Low ▪ Higher Ideological Differences


▪ Competition is Centripetal ▪ Competition is Centrifugal
▪ (Towards Centre) ▪ (Away from Centre)
▪ Coalitions are easily formed ▪ Coalitions are not viable

CENTRIPETAL FORCE CENTRIFUGAL FORCE


( TOWARDS CENTRE ) ( AWAY FROM CENTRE )
SOURCES

Source : Comparative Politics by Daniele Caramani . Pg. 240,241 .


DIRECT
NEW
CONCEPTS COMPARATIVE POLITICS

8. Which of the following principles are used to study the ‘attitudes of people’ i.e.
political culture in comparative politics?
A. Riker’s strategic principle
B. Converse principle
C. Robinson’s principle
D. Gallagher’s least square index
E. Rae’s fractionalization index
Choose the most appropriate answer from the options given below:
1 B and C only 2 C and E only

3 A and B only 4 B and D only


DIRECT
NEW
CONCEPTS COMPARATIVE POLITICS

8. Which of the following principles are used to study the ‘attitudes of people’ i.e.
political culture in comparative politics?
A. Riker’s strategic principle
B. Converse principle
C. Robinson’s principle
D. Gallagher’s least square index
E. Rae’s fractionalization index
Choose the most appropriate answer from the options given below:
1 B and C only 2 C and E only

3 A and B only 4 B and D only


EXPLANATION
▪ In Comparative Politics, there are certain principles which are important for understanding how political culture works.

▪ They help us to understand how people's attitudes are related to each other, and how those attitudes may differ at the
aggregate and individual levels.

▪ By comparing the values of people in different countries, we can learn about the different political cultures that exist
around the world. This information can be used to understand why countries have different political systems and why
they make different political decisions.

▪ Converse principle : It states that there is a high correlation between different value orientations. This means that people
who have similar views on one political issue are likely to have similar views on other political issues. For example, people
who believe in democracy are also likely to believe in individual rights and limited government.

▪ Robinson's principle : It states that macro values may deviate from micro values. This means that the values that people
express at the aggregate level (e.g., the national level) may not be the same as the values that people express at the
individual level. For example, a survey may find that a majority of people in a country support democracy, but when you
look at individual respondents, you may find that many people have contradictory views about democracy.
SOURCES

Source : Comparative Politics by Daniele Caramani . Pg. 424 .


DIRECT
NEW
THINKERS COMPARATIVE POLITICS

9. Match List I with List II :


List-I (Concept/Theory) List-II (Proponent)

A. Critique of rational choice institutionalism’s I. Campbell & Pederson


advocacy of universal generalization
B. Study of institutional change in skill regimes in II. Scharpf
Britain, Germany, Japan and the US
C. Origins of sociological institutionalism III. Kathleen Thelen
D. Discursive institutionalism IV. Meyer and Rowan

1 A-II, B-IV, C-I, D-III 2 A-III, B-IV, C-I, D-II


3 A-II, B-III, C-IV, D-I 4 A-I, B-II, C-III, D-IV
DIRECT
NEW
THINKERS COMPARATIVE POLITICS

9. Match List I with List II :


List-I (Concept/Theory) List-II (Proponent)

A. Critique of rational choice institutionalism’s I. Campbell & Pederson


advocacy of universal generalization
B. Study of institutional change in skill regimes in II. Scharpf
Britain, Germany, Japan and the US
C. Origins of sociological institutionalism III. Kathleen Thelen
D. Discursive institutionalism IV. Meyer and Rowan

1 A-II, B-IV, C-I, D-III 2 A-III, B-IV, C-I, D-II


3 A-II, B-III, C-IV, D-I 4 A-I, B-II, C-III, D-IV
EXPLANATION
▪ This question deals with various concepts / theories given by Comparative Scholars on various Types of New Institutionalism.

▪ A. “Critique of Rational Choice Institutionalism’s Advocacy of Universal Generalization” – by Fritz W. Scharpf.


▪ Firstly, Let us understand – What is Rational Choice institutionalism?
▪ It is a theoretical approach that examines how rational actors' interests and motivations influence their behavior within specific
institutional contexts.
▪ It uses deductive reasoning to understand actors' reasons for actions, predict likely outcomes, and identify institutional incentives.

▪ What is F.W. Scharpf’s criticism against Rational Choice Institutionalism?


▪ While Rational Choice theory is effective at capturing typical reasons for behavior, it struggles in two aspects –
▪ (1) Firstly, it fails to explain exceptions or irregularities in situations that deviate significantly from interest-driven actions.
▪ (2) Secondly, over-generalization is also an issue with Rational Choice Institutionalism because it aims for giving universal explanations.

▪ What is the Solution that F.W. Scharpf gives?


▪ On the other hand, Scharpf gives us the ‘Actor-Centered institutionalism’ approach. This theory avoids these “universal generalizing”
problems by employing 'bounded generalizations' to explain policy outcomes through factors like problems, policy legacies, actors'
attributes, and institutional interactions in specific cases, not universalizing all cases.
EXPLANATION
▪ B. “Study of institutional change in skill regimes in Britain, Germany, Japan and the US” – by Kathleen Thelen.
▪ Here, the context is on – Historical Institutionalism – the branch of New Institutionalism which focuses on – studying how political
institutions develop over time. According to this branch – Institutions are like regular patterns and routines that result from intentional
decisions and specific historical circumstances. This process in Historical Institutionalism is referred to as a 'logic of path-dependence,'
where past choices shape the current outcomes, often unintentionally.
▪ Now, Critics of this school argue that -
▪ (1) Historical Institutionalism may seem too focused on Historical determinism or Mechanistic thinking because it emphasizes
continuities and assumes that change happens only in sudden bursts, with periods of little to no change in between.
▪ (2) Historical Institutionalism give a lot of emphasis to structures and they fail to take into account the factor of human agency in
institutions.
▪ What is the Solution to these 2 problems ?
▪ To Correct the first problem, scholars like Kathleen Thelen and Wolfgang Streeck have focused on the process of Incremental changes in
institutions. They suggest that gradual changes can be more transformative than abrupt ones in institutions.
▪ To Correct the second problem, Kathleen Thelen conducts a mixed research study – where she combines elements of
Historical institutionalism with Rational choice institutionalism ( How Historical Structures + shape Actors’ Interests )
▪ Kathleen Thelen explores how skills systems changed in countries like Britain, Germany, Japan, and the United States.
Instead of only looking at historical patterns, she uses rationalist explanations that center around ongoing negotiations between
different political groups and conflicts. These interactions lead to changes in the systems through a process of layering (adding new
elements) and conversion (changing existing elements). In other words, Thelen studies how political groups working together or clashing
can bring about changes in the way skills systems function in these countries.
EXPLANATION
▪ C. “Origins of sociological institutionalism” – by Meyer and Rowan.
▪ Sociological institutionalism views political institutions as shaped by Society and Culture, where people in politics follow a
‘Logic of Appropriateness’ based on behavior guided by culturally fitting norms and expected social conduct.
▪ Although in Political Science, Sociological institutionalism was introduced by James March and Johan Olsen’s work
“Rediscovering Institutions” in 1989.
▪ The very first origins of Sociological institutionalism can be traced back to the writings of two organizational theorists –
John W. Meyer and Brian Rowan’s work “Institutionalized Organizations: Formal Structure as Myth and Ceremony” in 1977.

▪ D. “ Discursive institutionalism ” – Campbell & Pederson.


▪ Now in the late 1970s and early 1980s ‘New institutionalism’ began with the desire of a wide range of scholars to Bring Institutions back
into the explanation of Political action.
▪ From the one ‘new institutionalism’ originally defined in the 1980s by James March and Johan Olsen and then
three ‘new institutionalisms’ elaborated in the mid 1990s by Peter Hall and Rosemary Taylor (1996),
Vivien A. Schmidt argues we can now identify four main new institutionalist approaches: the three older ‘new institutionalisms’ –
Rational Choice, Historical, and Sociological institutionalism – plus a fourth newer ‘new institutionalism’ – Discursive institutionalism.
▪ What is Discursive institutionalism?
▪ Discursive institutionalism considers the ideas and discourse that actors use to devise, deliberate, and/or legitimize political action in
institutional context – whether seen as incentive structures, regularized practices, or social constructions – according to a ‘Logic of
Communication’ (the organized patterns and rules that guide how ideas are shared among individuals within institutions).
Thinkers of Discursive institutionalism – John L. Campbell, Ove K. Pedersen, Vivien A. Schmidt, Peter Kjaer.
EXPLANATION
FOUR TYPES OF NEW - INSTITUTIONALISM

Rational Choice Historical Sociological Discursive


Institutionalism Institutionalism Institutionalism Institutionalism

Can Explain how Rational Can Explain how Regular Can Explain how Political Explain how Ideas and
Actors pursue their Patterns and Routines are institutions are shaped by Discourse shape
preferences in institutions formed from intentional Cultural Political Action in
through decisions and specific Norms and Frames institutional context
‘Logic of Calculation’ historical circumstances through through
through ‘Logic of Appropriateness’ ‘Logic of Communication’
‘Logic of Path-dependency’

Thinkers – Thinkers – Thinkers – Thinkers –


Olsen, North, Blyth, Vargwese, James March, Johan Olsen, John L. Campbell,
Levi, Rothstein. Kathleen Thelen, Katzenstein, Meyer, Ove K. Pedersen,
Wolfgang Streeck Rowan, DiMaggio, Powell. Vivien A. Schmidt,
Peter Kjaer
SOURCES

Source : The SAGE Handbook of Comparative Politics. Pg . 127, 129, 130 , 132 .
HOW TO PREPARE FOR DECEMBER EXAM 2023 ?

▪ To FOCUS ON –
▪ Read Pointwise All the Arguments made by a Thinker
▪ Mark Keywords to Identify Unique aspect of Each Thinker
▪ Cover NOT Just Facts, Prepare also the in-depth Points

UNIT – 4
▪ To FOCUS ON –
▪ To Select One Very Good Quality Textbook for CP. Stick to
One Book and Read it Thoroughly (2x times).
COMPARATIVE ▪ To Prepare New Thinkers along with Classical CP thinkers
POLITICS
▪ Reference Material :-
▪ Oxford Comparative Politics by Daniele Caramani, or
▪ Theoretical Foundations of Comparative Politics by Sushila
Ramaswamy, or Eduseeker’s Ebook Rapid Fire Revision (CP).
INTERNATIONAL
RELATIONS
DIRECT
NTA
PYQs
INTERNATIONAL RELATIONS

1. Who among the following political thinkers reformulated the


principle of realism by Hans J. Morgenthau from a feminist
perspective?
1 Gunder Frank 2 Janet Abu

3 Ann Tickner 4 Hannes Peltonen


DIRECT
NTA
PYQs
INTERNATIONAL RELATIONS

1. Who among the following political thinkers reformulated the


principle of realism by Hans J. Morgenthau from a feminist
perspective?
1 Gunder Frank 2 Janet Abu

3 Ann Tickner 4 Hannes Peltonen


EXPLANATION
▪ Ann J. Tickner is a Feminist IR scholar who has written extensively on the gendered nature
of international relations theory. In her book Gendering World Politics, she reformulated
the Six principles of realism by Hans J. Morgenthau from a feminist perspective as follows :
▪ 1. Dynamic Objectivity – Human nature is both masculine and feminine. Objectivity is
culturally defined and it is associated with masculinity – so objectivity is always partial.
▪ 2. National Interest is Dynamic – It cannot be just defined in terms of Power alone.
It is more dynamic. The national interest is multi-dimensional – so it cannot
(and should not) be defined by one set of interests.
▪ 3. Power cannot be defined universally – Power in Realist lens was domination and
control. It ignores the possibility of ‘Collective Empowerment’.
▪ 4. Morality and Politics cannot be separated – All Political Actions have moral
significance behind it.
▪ 5. To Discover Moral Commonalities in Human.
▪ 6. Rejects Autonomy of Politics in IR – A narrowly defined, and ‘autonomous’,
political realm defines the political in a way that excludes the concerns and
contributions of women.
SOURCES

Source : Global Politics by Andrew Heywood. Pg. 109 .

Source : Rapid Fire Revision Ebook (Eduseeker) . Pg . 108 .


DIRECT
FIRST TIME
EVENTS INTERNATIONAL RELATIONS

2. Arrange the following peacekeeping operations in chronological order :


A. UNAMID in Darfur
B. UNMISS in South Sudan
C. UNAMIR in Rwanda
D. UNPROFOR in former Yugoslavia

Choose the correct answer from the options given below :

1 A, B, C and D 2 B, C, A and D

3 D, C, A and B 4 C, D, B and A
DIRECT
FIRST TIME
EVENTS INTERNATIONAL RELATIONS

2. Arrange the following peacekeeping operations in chronological order :


A. UNAMID in Darfur
B. UNMISS in South Sudan
C. UNAMIR in Rwanda
D. UNPROFOR in former Yugoslavia

Choose the correct answer from the options given below :

1 A, B, C and D 2 B, C, A and D

3 D, C, A and B 4 C, D, B and A
EXPLANATION

▪ The following UN Peacekeeping Operations arranged in chronological order :

➢ UNPROFOR in former Yugoslavia – Feb 1992 - Dec 1995.

➢ UNAMIR in Rwanda – Oct 1993 - March 1996.

➢ UNAMID in Darfur – July 2007 - Present.

➢ UNMISS in South Sudan – July 2011 - Present.


SOURCES

Source : Essentials of International Relations by Karen A. Mingst


and Heather Elko McKibben. Pg . 332 .
DIRECT
NTA
SYLLABUS INTERNATIONAL RELATIONS

3. The justification for humanitarian intervention lies in


A. Global interdependence
B. National interest
C. Moral pluralism
D. Regional stability

Choose the most appropriate answer from the options given below:

1 A and C only 2 B and C only

3 B and D only 4 A and D only


DIRECT
NTA
SYLLABUS INTERNATIONAL RELATIONS

3. The justification for humanitarian intervention lies in


A. Global interdependence
B. National interest
C. Moral pluralism
D. Regional stability

Choose the most appropriate answer from the options given below:

1 A and C only 2 B and C only

3 B and D only 4 A and D only


EXPLANATION
▪ Humanitarian intervention is a form of military intervention that is carried out in pursuit of humanitarian rather than
strategic objectives. It is one of the most hotly disputed issues in global politics. While some see it as evidence that world
affairs are being guided by new and more enlightened cosmopolitan sensibilities, others view humanitarian intervention
as deeply misguided and morally confused.

▪ There are Two points in the Question that justify in favour of Humanitarian intervention and two points that cannot
justify the cause of Humanitarian intervention.

▪ The following Two points justify in favour of Humanitarian intervention :-


▪ Global Interdependence – As the world becomes more connected and dependent on each other, there's a greater
responsibility to respond to events happening far away. Countries can't act like they're all alone anymore. Helping in
crises in other places can make sense for their own benefit too. Humanitarian intervention can therefore be justified
on grounds of enlightened self-interest. For instance, stopping a refugee crisis can prevent problems in other
countries and benefit us in the long run.
▪ Regional Stability – When there's a big problem like a humanitarian crisis, especially if a country is falling apart, it
can really mess up the power balance in that area. This makes things unstable and can cause even bigger problems.
Because of this, nearby countries might want to help out to stop things from getting worse. Even bigger countries
might step in to prevent a big war from happening in that whole region.
EXPLANATION
▪ The following Two points does not justify the cause of Humanitarian intervention :-

▪ National Interests – Realists argue that countries only care about their own benefits, so when they say they're using
military force for humanitarian reasons, they're probably lying for political reasons. On the other hand, if an
intervention were genuinely humanitarian, the state engaging in humanitarian question would be putting its own
citizens at risk to ‘save strangers’, violating its own national interests of protecting its own people.

▪ Moral Pluralism – Humanitarian intervention can be seen as a form of cultural imperialism, in that it is based on an
essentially Western notion of human rights that may not be applicable in other parts of the world. There are deep
historical, cultural, and religious differences which makes it impossible to establish universal guidelines for the
behaviour of governments, it's hard to just set one set of rules that everyone should follow. This makes it really
tough to decide when it's okay to intervene in a situation.
SOURCES

Arguments in
FAVOUR of
Humanitarian
Intervention

Arguments
AGAINST
Humanitarian
Intervention

Source : Global Politics by Andrew Heywood. Pg. 427 .


DIRECT
NEW
SYLLABUS INTERNATIONAL RELATIONS

4. Match List I with List II :


List-I (Thinkers) List-II (Concepts)

A. Robert Cox I. Impact of international regimes on cooperation


B. Robert Keohane II. Interaction between social forces, forms of state and world orders.
C. Alexander Wendt III. Construction idea of social structures
D. Rob Walker IV. Emphasis on continuity and repetition in social structures.

1 A-II, B-I, C-III, D-IV 2 A-I, B-II, C-III, D-IV


3 A-III, B-IV, C-I, D-II 4 A-II, B-I, C-IV, D-III
DIRECT
NEW
SYLLABUS INTERNATIONAL RELATIONS

4. Match List I with List II :


List-I (Thinkers) List-II (Concepts)

A. Robert Cox I. Impact of international regimes on cooperation


B. Robert Keohane II. Interaction between social forces, forms of state and world orders.
C. Alexander Wendt III. Construction idea of social structures
D. Rob Walker IV. Emphasis on continuity and repetition in social structures.

1 A-II, B-I, C-III, D-IV 2 A-I, B-II, C-III, D-IV


3 A-III, B-IV, C-I, D-II 4 A-II, B-I, C-IV, D-III
EXPLANATION
▪ These are Four Concepts described by different IR Scholars :-

▪ Robert Cox – “Interaction between social forces, forms of state and world orders” – Robert Cox theorizes that there is a
complex and dynamic relationship between three elements: social forces, forms of state, and world orders.
▪ The term ‘social forces’ is shorthand for the process of capitalist production.
▪ ‘Forms of state’ point to the ways in which states change in the interplay with the social forces of capitalist development.
▪ The term ‘world orders’ refers to the current organization of international relations, including relations between major
states and groups of states, the status of international law, and international institutions.
▪ These elements are intricately linked and influence one another.
▪ Robert Keohane – “Impact of international regimes on cooperation” – Keohane's main point is that international
cooperation doesn't solely depend on the presence of a hegemonic power.
▪ Instead, he argues that when countries have shared goals and interests, they can establish international regimes or
agreements that outline rules and guidelines for cooperation.
▪ Once these international institutions are set up, they continue to function independently, regardless of the states that
created them. These regimes have their own staying power and can continue to encourage cooperation among countries
even in situations where a hegemonic power is declining.
EXPLANATION
▪ Alexander Wendt – “Construction idea of social structures” – Wendt refers to the concept that the social structures
within a society, including institutions, systems, and relationships, are not inherent or naturally existing entities,
but rather they are shaped and brought into existence through human thought processes, beliefs, and interpretations.

▪ It emphasizes that these social structures are not merely the result of physical entities or material resources, but rather
they are formed and organized based on the ideas, concepts, and understandings that people have about these entities.

▪ Rob Walker – “Emphasis on continuity and repetition in social structures” – This quote refers to the criticism posed by
post-structuralist thinkers against neorealism. By highlighting the concept of "continuity and repetition,"
Rob Walker emphasizes how neorealism fails to account for the dynamic nature of social systems and their
potential for change.

▪ This perspective results in treating individual actors as passive entities bound to perpetuate existing structures, and it
prevents neorealism from effectively addressing the transformations and developments in international relations. The
criticism highlights the limitation of neorealism's ability to adapt to changing circumstances and explore alternative
possibilities beyond the rigid dichotomies it presents.
SOURCES

Source : Introduction to International Relations by Robert Jackson, Georg Sørensen, Jørgen Møller. Pg . 280,184,195,226 .
DIRECT
NTA
SYLLABUS
INTERNATIONAL RELATIONS

5. The idealist school of thought in international relations proposed to


replace ‘balance of power’ by -

1 Secret diplomacy 2 Disarmament

3 Autocratic rules 4 War


DIRECT
NTA
SYLLABUS
INTERNATIONAL RELATIONS

5. The idealist school of thought in international relations proposed to


replace ‘balance of power’ by -

1 Secret diplomacy 2 Disarmament

3 Autocratic rules 4 War


EXPLANATION

▪ The Idealist school of thought in international relations is based on the belief that international cooperation
and understanding can be achieved through the promotion of shared values and ideals, such as democracy,
human rights, and the rule of law.

▪ Idealists believe that the balance of power is a flawed system that leads to conflict and instability. They argue
that disarmament is a more effective way to ensure peace and security.

▪ Idealists argue that the balance of power is inherently unstable. As the distribution of power changes, it can
lead to shifts in alliances and alignments, which can increase the risk of conflict.

▪ Idealists argue that Disarmament can help to build trust and cooperation between states. When states
disarm, they are less likely to see each other as a threat, and they are more likely to work together to solve
common problems.
DIRECT
FIRST TIME
COMMISSION
INTERNATIONAL RELATIONS

6. The commission on the Status of Women was established vide


resolution of the –

1 ECOSOC 2 UNEP

3 WHO 4 UNEF
DIRECT
FIRST TIME
COMMISSION
INTERNATIONAL RELATIONS

6. The commission on the Status of Women was established vide


resolution of the –

1 ECOSOC 2 UNEP

3 WHO 4 UNEF
EXPLANATION

▪ The Commission on the Status of Women (CSW) is the principal global intergovernmental
body exclusively dedicated to the promotion of gender equality and the empowerment of
women.

▪ It was established by ECOSOC – Resolution 11(II) on 21 June 1946.

▪ The Commission on the Status of Women is instrumental in promoting women’s rights,


documenting the reality of women’s lives throughout the world, and shaping global
standards on gender equality and the empowerment of women.
SOURCES

Source : Essentials of International Relations by Karen A. Mingst


and Heather Elko McKibben. Pg . 329 .

Source : UN Women. Commission on the Status of Women.


DIRECT
NTA
PYQs
INTERNATIONAL RELATIONS

7. Who wrote the book ‘State in capitalist society’?

1 Nicolas Poulantzas 2 Karl Marx

3 Rosa Luxemburg 4 Ralph Miliband


DIRECT
NTA
PYQs
INTERNATIONAL RELATIONS

7. Who wrote the book ‘State in capitalist society’?

1 Nicolas Poulantzas 2 Karl Marx

3 Rosa Luxemburg 4 Ralph Miliband


EXPLANATION
▪ The book ‘State in Capitalist Society’ written by Ralph Miliband – the Instrumentalist thinker.
▪ The instrumentalist view of the state is a Marxist theory that argues
the State is an instrument of class rule by the capitalist class.
This theory was first developed by Ralph Miliband in this book.
▪ The book sheds light on how the state, which was initially hidden behind the complexities of the political
system, has become a central topic in political studies.
▪ Miliband argues that the radical alternative of adopting socialism is the key issue facing civilization.
▪ Miliband shows us how capitalist control over the state is so extensive that making even only partial reforms
also becomes impossible.
▪ Miliband also explains how society has managed to evade socialism, how the claims of Socialism have failed
to persuade many intellectuals and the potential benefactors of an alternative order.
▪ The book delves into the influence of economic elites and the dominant class in shaping the state's actions.
It questions the legitimacy of the state's claims and examines the role and purpose of governments.
The concepts of reform and repression are analyzed as well.
▪ Miliband argues that the state uses its power to maintain the capitalist system and to protect the interests of
the capitalist class.
SOURCES

Source : Rapid Fire Revision Ebook (Eduseeker) . Pg . 111 .


DIRECT
NTA
SYLLABUS INTERNATIONAL RELATIONS

8. Which among the following statements represents the principles 8. निम्ननिखित में से कौि सा कथि न्याय संगत युद्ध के नसद्धांत ं का
of Just War? प्रनतनिनित्व करता है ?
A. War as the last resort A. अंनतम उपाय के रूप में युद्ध ।
B. The purpose of war is revenge B. युद्ध का प्रय जि प्रनतश ि है।
C. War must be declared on the basis of normally acceptable aims C. युद्ध अनिवाययत: िैनतक रूप से स्वीकायय िक्ष् ं के आिार पर ही
घ नित नकया जािा चानहए।
D. A wholesale invasion is a justifiable response to a border
incursion D. युद्ध के फिस्वरूप हानि से अनिक िाभ ह िा चानहए।
E. ??? E. समग्रत: बडे स्तर पर आक्रमण सीमा पर ह िे वािे आक्रमण की
न्याय नचत अिुनक्रया है।
Choose the most appropriate answer from the questions given
below: िीचे निए गए नवकल् ं में से सही उत्तर का चयि कीनजए:

1 A, C and D only 2 A, B and C only

3 A and C only 4 A, C, D and E only


DIRECT
NTA
SYLLABUS INTERNATIONAL RELATIONS

8. Which among the following statements represents the principles 8. निम्ननिखित में से कौि सा कथि न्याय संगत युद्ध के नसद्धांत ं का
of Just War? प्रनतनिनित्व करता है ?
A. War as the last resort A. अंनतम उपाय के रूप में युद्ध ।
B. The purpose of war is revenge B. युद्ध का प्रय जि प्रनतश ि है।
C. War must be declared on the basis of normally acceptable aims C. युद्ध अनिवाययत: िैनतक रूप से स्वीकायय िक्ष् ं के आिार पर ही
घ नित नकया जािा चानहए।
D. The outcome of war should be more benefit than loss.
D. युद्ध के फिस्वरूप हानि से अनिक िाभ ह िा चानहए।
E. A wholesale invasion is a justifiable response to a border
incursion E. समग्रत: बडे स्तर पर आक्रमण सीमा पर ह िे वािे आक्रमण की
न्याय नचत अिुनक्रया है।
Choose the most appropriate answer from the questions given
below: िीचे निए गए नवकल् ं में से सही उत्तर का चयि कीनजए:

1 A, C and D only 2 A, B and C only

3 A and C only 4 A, C, D and E only


DIRECT
NTA
SYLLABUS INTERNATIONAL RELATIONS

8. Which among the following statements represents the principles 8. निम्ननिखित में से कौि सा कथि न्याय संगत युद्ध के नसद्धांत ं का
of Just War? प्रनतनिनित्व करता है ?
A. War as the last resort A. अंनतम उपाय के रूप में युद्ध ।
B. The purpose of war is revenge B. युद्ध का प्रय जि प्रनतश ि है।
C. War must be declared on the basis of normally acceptable aims C. युद्ध अनिवाययत: िैनतक रूप से स्वीकायय िक्ष् ं के आिार पर ही
घ नित नकया जािा चानहए।
D. The outcome of war should be more benefit than loss.
D. युद्ध के फिस्वरूप हानि से अनिक िाभ ह िा चानहए।
E. A wholesale invasion is a justifiable response to a border
incursion E. समग्रत: बडे स्तर पर आक्रमण सीमा पर ह िे वािे आक्रमण की
न्याय नचत अिुनक्रया है।
Choose the most appropriate answer from the questions given
below: िीचे निए गए नवकल् ं में से सही उत्तर का चयि कीनजए:

1 A, C and D only 2 A, B and C only

3 A and C only 4 A, C, D and E only


EXPLANATION
▪ There are 3 Major Issues with this Question.
▪ Issue No. 1 – NTA has mentioned 4 Options in English Version Question and 5 Options in Hindi Version Question.
▪ Issue No. 2 – Option D of English Version is Option E of Hindi Version.
▪ Issue No. 3 – Option E of English Version is missing itself.
▪ Therefore, This creates a major confusion as to why the Answer should be A, C, D.
If we take into account the Hindi Version of the Question = then answer is = A, C, D.
If we take into account the English Version of the Question (with Four statements A to D only) = then answer is = A and C.

▪ Now let us see which statements are correct and incorrect – in the context of Principles of Just War :-
▪ These Statements are True –
“War as the last resort.”
“War must be declared on the basis of normally acceptable aims.”
“The outcome of war should be more benefit than loss.”
▪ These Statements are False –
“The purpose of war is revenge.”
“A wholesale invasion is a justifiable response to a border incursion.”
EXPLANATION
▪ In the context of Principles of Just War :-
▪ These Statements are True –
▪ “War as the last resort.” – All non-violent options must have been exhausted before force can be justified. This is
sometimes seen as the principle of necessity.
▪ “War must be declared on the basis of normally acceptable aims.” – This principle is called Right intention - War must be
prosecuted on the basis of aims that are morally acceptable rather than revenge or the desire to inflict harm.
▪ “The outcome of war should be more benefit than loss.” – There should be a reasonable prospect of success.
War should not be fought in a hopeless cause, in which life is expended for no purpose or benefit.

▪ These Statements are False –


▪ “The purpose of war is revenge.” – This is false because War must be prosecuted on the basis of aims that are morally
acceptable rather than revenge or the desire to inflict harm.
▪ “A wholesale invasion is a justifiable response to a border incursion.” – This principle is called Principle of Proportionality.
War should result in more good than evil. It argues that any response to an attack in war should be measured and
proportionate. A wholesale invasion is not a justifiable response to a border incursion. For Example :- India-China border
incursions do not always result in wholesale invasions.
SOURCES

Source : Global Politics by Andrew Heywood . Pg. 257 .


DIRECT
NTA
SYLLABUS INTERNATIONAL RELATIONS

9. Match List I with List II :


List-I (SDG Goal Number) List-II (Goals)
A. 5 I. Affordable and clean energy
B. 7 II. Life on land
C. 10 III. Gender equity
D. 15 IV. Reduced inequalities

1 A-I, B-II, C-III, D-IV 2 A-II, B-I, C-IV, D-III


3 A-III, B-I, C-IV, D-II 4 A-I, B-III, C-II, D-IV
DIRECT
NTA
SYLLABUS INTERNATIONAL RELATIONS

9. Match List I with List II :


List-I (SDG Goal Number) List-II (Goals)
A. 5 I. Affordable and clean energy
B. 7 II. Life on land
C. 10 III. Gender equity
D. 15 IV. Reduced inequalities

1 A-I, B-II, C-III, D-IV 2 A-II, B-I, C-IV, D-III


3 A-III, B-I, C-IV, D-II 4 A-I, B-III, C-II, D-IV
EXPLANATION
SDG 1 NO POVERTY.
SDG 2 ZERO HUNGER.
SDG 3 GOOD HEALTH AND WELL-BEING.
SDG 4 QUALITY EDUCATION.
SDG 5 GENDER EQUALITY.
SDG 6 CLEAN WATER AND SANITATION.
SDG 7 AFFORDABLE AND CLEAN ENERGY.
SDG 8 DOESN’T WORK AND ECONOMIC GROWTH.
SDG 9 INDUSTRY, INNOVATION, AND INFRASTRUCTURE.
SDG 10 REDUCED INEQUALITIES.
SDG 11 SUSTAINABLE CITIES AND COMMUNITIES.
SDG 12 RESPONSIBLE CONSUMPTION AND PRODUCTION.
SDG 13 CLIMATE ACTION.
SDG 14 LIFE BELOW WATER.
SDG 15 LIFE ON LAND.
SDG 16 PEACE, JUSTICE, AND STRONG ISNTITUTIONS.
SDG 17 PARTNERSHIPS FOR THE GOALS.
SOURCES

Source : Sustainable Development Goals (17)


DIRECT
NTA
SYLLABUS INTERNATIONAL RELATIONS

10. Which of the following statement represent the impact of Globalisation on Terrorism?
A. Globalisation has permitted exchange of ideas and goods which is exploited by terrorist
groups.
B. The internet has allowed terrorists to recruit new followers.
C. New technologies allow terrorists to operate in highly distributed network.
D. Insurgent and terrorist groups in Iraq and Syria have not built sophisticated improvised
explosive devices (IEDs) despite global access to resources.
Choose the most appropriate answer from the options given below:
1 A, B, C and D only 2 B and C only

3 A, B, C only 4 A, B, D only
DIRECT
NTA
SYLLABUS INTERNATIONAL RELATIONS

10. Which of the following statement represent the impact of Globalisation on Terrorism?
A. Globalisation has permitted exchange of ideas and goods which is exploited by terrorist
groups.
B. The internet has allowed terrorists to recruit new followers.
C. New technologies allow terrorists to operate in highly distributed network.
D. Insurgent and terrorist groups in Iraq and Syria have not built sophisticated improvised
explosive devices (IEDs) despite global access to resources.
Choose the most appropriate answer from the options given below:
1 A, B, C and D only 2 B and C only

3 A, B, C only 4 A, B, D only
EXPLANATION

▪ The Statements A, B, C are true in the context of Globalisation on Terrorism :-


▪ A. Elements of globalization that permit the rapid exchange of ideas and goods are also exploited by terrorist
groups.
▪ B. The internet and social media allow terrorists to reach and influence audiences instantaneously and
recruit new followers.
▪ C. The technologies associated with globalization allow terrorists to operate in a highly distributed global
‘network’ to share information, conduct highly coordinated, lethal attacks, and permit a high degree of
mobility and security.
▪ While Statement D is false.
▪ Globalization has facilitated access to the weapons and resources required to conduct smaller, but more
lethal attacks. In Iraq, Syria, and Afghanistan, insurgent and terrorist groups have built sophisticated IEDs.
IEDs stand for - ‘improvised explosive devices’.
▪ State sponsorship may no longer be required for the construction of IEDs given the broad underworld
networks formed as a result of globalization.
SOURCES

Source : The Globalization of World Politics by Baylis and Smith. Pg. 460 .
DIRECT
NEW
FACTS
INTERNATIONAL RELATIONS

11. Which of the following agreements does not outline country-


specific emission targets but emphasizes on climate change
mitigation?
1 Kyoto Protocol 2 Law of Sea Treaty

3 Montreal Protocol 4 The Paris Agreement


DIRECT
NEW
FACTS
INTERNATIONAL RELATIONS

11. Which of the following agreements does not outline country-


specific emission targets but emphasizes on climate change
mitigation?
1 Kyoto Protocol 2 Law of Sea Treaty

3 Montreal Protocol 4 The Paris Agreement


EXPLANATION

▪ The climate agreement that does not outline country-specific emission targets but emphasizes climate change mitigation
is the 2015 Paris Agreement.

▪ The Paris Agreement was adopted in 2015 under the United Nations Framework Convention on Climate Change
(UNFCCC). It aims to limit global warming to well below 2 degrees Celsius above pre-industrial levels, with efforts to limit
it to 1.5 degrees Celsius.

▪ While the Paris Agreement requires all participating countries to submit nationally determined contributions (NDCs)
outlining their climate actions and targets, it does not enforce specific emission reduction targets for individual countries.

▪ Instead, it operates on a bottom-up approach, allowing each country to set its own goals based on its capabilities and
circumstances.

▪ This flexibility was designed to encourage broader global participation and engagement in the effort to combat climate
change.
SOURCES

Source : Introduction to International Relations by Robert Jackson, Georg Sørensen, Jørgen Møller. Pg . 318.
DIRECT
FIRST TIME
ECO. EVENTS INTERNATIONAL RELATIONS

12. Chronologically arrange the moments of Crises of Modern Capitalism :


A. U.S. sub-prime mortgage crisis
B. Argentine economic crisis
C. Dot com crisis in IT- related stocks
D. Mexican economic crisis

Choose the correct answer from the options given below :

1 D, B, C and A 2 B, D, A and C

3 A, B, C and D 4 D, B, A and C
DIRECT
FIRST TIME
ECO. EVENTS INTERNATIONAL RELATIONS

12. Chronologically arrange the moments of Crises of Modern Capitalism :


A. U.S. sub-prime mortgage crisis
B. Argentine economic crisis
C. Dot com crisis in IT- related stocks
D. Mexican economic crisis

Choose the correct answer from the options given below :

1 D, B, C and A 2 B, D, A and C

3 A, B, C and D 4 D, B, A and C
EXPLANATION

▪ Major Financial Crises of Modern Capitalism arranged in chronological order :

➢ Mexican economic crisis – 1994-95.

➢ Argentine economic crisis – 1999-2002.

➢ Dot-com crisis – 2000.

➢ U.S. sub-prime mortgage crisis – 2007-08.


SOURCES

Source : Global Politics by Andrew Heywood . Pg. 106 .


SOURCES

Source : ALL India JRF-NET Political Science FREE MOCK TEST.


DIRECT
NTA
SYLLABUS INTERNATIONAL RELATIONS

13. Assertion (A): Economic globalisation promotes prosperity and


opportunity for all.

Reason (R): Economic globalisation promotes an open, market-based


economy that brings social and political benefits.

1 Both A and R are true and R is the correct explanation of A


2 Both A and R are true but R is NOT the correct explanation of A
3 A is correct but Statement R is false
4 A is not correct but Statement R is true
DIRECT
NTA
SYLLABUS INTERNATIONAL RELATIONS

13. Assertion (A): Economic globalisation promotes prosperity and


opportunity for all.

Reason (R): Economic globalisation promotes an open, market-based


economy that brings social and political benefits.

1 Both A and R are true and R is the correct explanation of A


2 Both A and R are true but R is NOT the correct explanation of A
3 A is correct but Statement R is false
4 A is not correct but Statement R is true
EXPLANATION

▪ Assertion (A) – Economic globalization refers to the interconnectedness and integration of economies around
the world through the movement of goods, services, capital, technology, and information across borders.
The argument here is that when economies are interconnected, it can lead to greater efficiency,
specialization, and increased access to markets. This, in turn, can potentially increase economic growth,
create jobs, and improve living standards, contributing to prosperity and opportunities for a broader range of
people.

▪ Reasoning (R) – A market-based economy relies on supply and demand forces to determine prices and
allocate resources, which can lead to increased efficiency, innovation, and competition. The reason further
claims that this open, market-based economy brings social and political benefits. These conditions promotes
prosperity and opportunity for all.

▪ Hence, Both A and R are true but R is the correct explanation of A.


SOURCES

Source : Introduction to International Relations by Robert Jackson, Georg Sørensen, Jørgen Møller. Pg . 297.
HOW TO PREPARE FOR DECEMBER EXAM 2023 ?

▪ To FOCUS ON –
▪ Cover Timelines and Key Concepts from A. Heywood
▪ Cover Featured Figures, FOCUS ON Boxes from A. Heywood

▪ To FOCUS ON –
UNIT – 5 ▪ Revise One Year’s Current Affairs in IR – Related to
Agreements, Conflicts, Summits.
▪ To Stick with One Good Textbook in IR and Update
INTERNATIONAL Yourself with Last 10 Year PYQs knowledge.

RELATIONS
▪ Reference Material :- (Any One Textbook + Guidebook)
▪ Global Politics by Andrew Heywood (3rd Edition).
▪ Globalization of World Politics by Baylis and Smith.
▪ Intro to International Relations by Jackson & Sørensen.
▪ Eduseeker’s International Relations Detailed IR Book.
▪ REMEMBER :- GET THE LATEST EDITION OF IR BOOKS.
INDIA’S
FOREIGN
POLICY
DIRECT
FIRST TIME
THINKERS INDIA’S FOREIGN POLICY

1. Match List I with List II :


List-I (Thinker) List-II (Determinates of Foreign Policy)
A. K. J. Holsti I. National role conceptions
B. Jonathan Mercer II. Emotion & Cognition co-produce emotions
C. Janice Gross Stein III. Process of Political learning
D. Arnold Wolfers IV. National security as ‘ambiguous symbol’

1 A-I, B-II, C-III, D-IV 2 A-II, B-III, C-IV, D-I


3 A-III, B-IV, C-I, D-II 4 A-IV, B-III, C-II, D-I
DIRECT
FIRST TIME
THINKERS INDIA’S FOREIGN POLICY

1. Match List I with List II :


List-I (Thinker) List-II (Determinates of Foreign Policy)
A. K. J. Holsti I. National role conceptions
B. Jonathan Mercer II. Emotion & Cognition co-produce emotions
C. Janice Gross Stein III. Process of Political learning
D. Arnold Wolfers IV. National security as ‘ambiguous symbol’

1 A-I, B-II, C-III, D-IV 2 A-II, B-III, C-IV, D-I


3 A-III, B-IV, C-I, D-II 4 A-IV, B-III, C-II, D-I
EXPLANATION
▪ The following are certain phrases or idioms which IR theorists have come up with to theorize on
Determinates of Foreign Policy :
▪ K. J. Holsti – ‘National Role conceptions’ – K. J. Holsti places an emphasis on ‘national role conceptions’.
Holsti focuses on how countries see their own roles. He suggests that it's important to recognize various
ways countries see their roles and not wrongly group together the foreign policies of all non-aligned nations
as if they're the same.
▪ Jonathan Mercer – ‘Emotion & Cognition co-produce Emotions’ – Including emotions in foreign policy
decisions, along with beliefs, is a new aspect to study the rationale behind foreign policy decision making.
Jonathan Mercer suggests that we shouldn't just think emotions come after thoughts or disrupt rationality;
instead, we can study how emotions and thoughts together create beliefs.
▪ Janice Gross Stein - ‘Process of Political learning’ – In Janice Gross Stein's view of political learning, if we
believe that knowledge is socially constructed, it is politics which determines knowledge and learning, it is
politics which influences what we know and how we learn. On the other hand, if we see knowledge as logical
truth, then what we learn impacts how politics work. Both reinforce each other.
▪ Janice Stein writes the case of Soviet leader Gorbachev, about how important political learning was as a
component of a broader explanation of the changes in Soviet foreign policy in the late 1980s.
EXPLANATION
▪ Arnold Wolfers - National security as ‘ambiguous symbol’ – Wolfers argue that realists, whether they’re
political leaders, experts, or scholars, often claim that the foreign policies they support are driven by what's
best for the country, especially they cite their policies in terms of 'National Security’.

▪ This makes sense given the importance of security in today's world. However, when terms like
“National Interest" or “National Security" become popular, they need careful examination. They might not
mean the same thing to everyone and might lack a clear definition. This popularity can lead to different
people using these terms to label their preferred policies, even if the labels are misleading.

▪ Arnold Wolfers points out that urging foreign policy based on national security interest is just as unclear and
deceptive as describing past actions. For such advice to be meaningful, it should specify how much security a
nation aims for and the methods to achieve it in a given situation. Sometimes it's wise to suggest more effort
and weapons, but in other cases, moderation and non-coercive methods might be better.

▪ Public opinions can easily swing from being overly confident to very concerned, so it's important to be
cautious of any simple solution, even one dressed up as realist policy focused solely on national security.
SOURCES

Source : The Oxford Handbook of Indian Foreign Policy edited by David Malone, C. Raja Mohan, Srinath Raghavan. Pg. 37,38.
DIRECT
FIRST TIME
CONCEPTS INDIA’S FOREIGN POLICY

2. Which among the following statements is true about rise of alternative


perspectives on Indian foreign policy?
A. In the mid-1960s, the Swatantra Party presented a classical conservative
perspective on foreign policy.
B. It was led by I.K. Gujral.
C. The supporters were drawn from former bureaucrats, business community
and members of princely families.
D. They united in their opposition to Nehru’s Fabian economies.
E. They believed in the forces of the market and private enterprises.
Choose the most appropriate answer from the questions given below:
1 A, B, C, D only 2 A, C, D, E only

3 B, C, D, E only 4 A, B, D, E only
DIRECT
FIRST TIME
CONCEPTS INDIA’S FOREIGN POLICY

2. Which among the following statements is true about rise of alternative


perspectives on Indian foreign policy?
A. In the mid-1960s, the Swatantra Party presented a classical conservative
perspective on foreign policy.
B. It was led by I.K. Gujral.
C. The supporters were drawn from former bureaucrats, business community
and members of princely families.
D. They united in their opposition to Nehru’s Fabian economies.
E. They believed in the forces of the market and private enterprises.
Choose the most appropriate answer from the questions given below:
1 A, B, C, D only 2 A, C, D, E only

3 B, C, D, E only 4 A, B, D, E only
EXPLANATION
▪ Stephen Cohen in his book India – Emerging Power, discusses on the topic of India’s Strategic thought.
▪ Cohen says that after the breakdown of Nehruvian consensus in India’s foreign policy, there has been a significant rise of
Alternative perspectives on India’s security and foreign policy.
▪ There are two major branches of this Alternative perspectives :
▪ (a) Conservative-Realists. (from the Swatantra party)
▪ (b) Ideologically-Revitalist perspective. (from the RSS and supporters of Hindutva ideology)

▪ Now, in the context of the Question asked by NTA, Statements A, C, D, E are True
with regards to “Conservative-Realists” perspectives on India’s foreign policy.
▪ In the mid-1960s, the Swatantra party attempted to offer a classical conservative perspective on domestic and foreign
policy issues.
▪ It was led by the veteran Congress politician C. Rajagopalachari and - NOT by IK Gujral.
▪ Swatantra’s supporters were drawn from three major groups: former government bureaucrats, members of princely
families, and the business community.
▪ What united these disparate factions was a distaste for Nehru’s Fabian economics and nonalignment; they were more
sympathetic to the market and to private enterprise and were strong anticommunists.
SOURCES

Source : India: Emerging Power by Stephen P. Cohen . Pg . 43 .


DIRECT
FIRST TIME
FACTS INDIA’S FOREIGN POLICY

3. Which among the following constitutes membership criteria of the NAM?


A. The country should support national independence movements.
B. The country should be a member of multilateral military alliance concluded
in the context of great power conflicts.
C. The country should have adopted an independent policy based on coexistence of
states.
D. If the country has granted military bases to a foreign power, the concession should be
in the context of great power conflicts.
Choose the most appropriate answer from the options given below:

1 A and B only 2 A and C only

3 A and D only 4 B, C and D only


DIRECT
FIRST TIME
FACTS INDIA’S FOREIGN POLICY

3. Which among the following constitutes membership criteria of the NAM?


A. The country should support national independence movements.
B. The country should be a member of multilateral military alliance concluded
in the context of great power conflicts.
C. The country should have adopted an independent policy based on coexistence of
states.
D. If the country has granted military bases to a foreign power, the concession should be
in the context of great power conflicts.
Choose the most appropriate answer from the options given below:

1 A and B only 2 A and C only

3 A and D only 4 B, C and D only


EXPLANATION
▪ There are 5 Criteria Points for gaining Membership of the Non-Aligned Movement (NAM) Countries, they are:
▪ 1. The Country should have adopted an independent policy based on the coexistence of States with different
political and social systems and on nonalignment, or it should have shown a tendency to favor such a policy.

▪ 2. The Country in question should support national independence movements in a consistent manner.

▪ 3. The Country should NOT be a member of a multilateral military alliance concluded in the context of great
power conflicts.

▪ 4. If the Country has a bilateral military agreement with a great power or it is a member of a regional defense
pact, the agreement or pact should have not been concluded deliberately in the context of great power
conflicts.

▪ 5. If the Country has granted military bases to a foreign power, the concession should have NOT been made
in the context of great power conflicts.
SOURCES

Source : Members and other Participants of NAM Movement. Ministry of External Affairs.
Link : https://www.mea.gov.in/Images/pdf/Members-and-other-participants.pdf
DIRECT
NTA
SYLLABUS INDIA’S FOREIGN POLICY

4. Statement I: India’s foreign remains pre-occupied with concerns about territorial


integrity, nationhood and independence of decision making.

Statement II : The structural logic of the global bipolar conflict gets reflected in the
regional bipolar conflict between India and Pakistan.

1 Both Statement I and Statement II are true


2 Both Statement I and Statement II are false
3 Statement I is correct but Statement Il is false
4 Statement I is incorrect but Statement II is true
DIRECT
NTA
SYLLABUS INDIA’S FOREIGN POLICY

4. Statement I: India’s foreign remains pre-occupied with concerns about territorial


integrity, nationhood and independence of decision making.

Statement II : The structural logic of the global bipolar conflict gets reflected in the
regional bipolar conflict between India and Pakistan.

1 Both Statement I and Statement II are true


2 Both Statement I and Statement II are false
3 Statement I is correct but Statement Il is false
4 Statement I is incorrect but Statement II is true
EXPLANATION
▪ The statements are made in the context of India’s conflicts with Pakistan, China, and the United States and
how they can be understood through the prism of five interpretive lenses :
➢ (1) Post-Colonial Sovereignty lens
(2) Alliance Pressures lens
(3) Power Distribution lens
(4) Conflict over Political Values lens
(5) Domestic Politics lens.
▪ Statement I – This statement refers to the Post-colonial sovereignty lens – The statement indicates that when
we study India’s quarrels vis-a-vis Pakistan, China, and the United States there are mainly three factors which
shape our foreign policy understanding – territorial integrity, nationhood, and independent decision making.
▪ Statement II – This statement refers to the Alliance Pressure lens – The statement refers to the idea that the
fundamental characteristics and dynamics of the larger global conflict between two superpowers (of the
United States and the Soviet Union during the Cold War) were mirrored or mirrored to a certain extent in the
regional conflict between India and Pakistan. It affected how Pakistan got closer to United States and India
while maintaining its non-aligned stance maintained closer ties with the Soviet Union.
▪ Hence, Both Statement I and Statement II are correct.
SOURCES

Source : The Oxford Handbook of Indian Foreign Policy edited by David Malone, C. Raja Mohan, Srinath Raghavan. Pg. 23,25.
DIRECT
FIRST TIME
FACTS INDIA’S FOREIGN POLICY

5. Which of the following is not included in the list of India’s initiatives in support of the
US in strategic collaboration since 2001?
A. India did not offer support for US anti-terrorism mission in Afghanistan.
B. India expressed opposition to President Bush’s decision to withdraw from the ABM
Treaty.
C. India voted with the US at September 2005 International Atomic Energy Compliance
Board of Governors meeting to declare that Iran was non-compliant of NPT.
Choose the most appropriate answer from the options given below:

1 A only 2 B only

3 C only 4 A and B only


DIRECT
FIRST TIME
FACTS INDIA’S FOREIGN POLICY

5. Which of the following is not included in the list of India’s initiatives in support of the
US in strategic collaboration since 2001?
A. India did not offer support for US anti-terrorism mission in Afghanistan.
B. India expressed opposition to President Bush’s decision to withdraw from the ABM
Treaty.
C. India voted with the US at September 2005 International Atomic Energy Compliance
Board of Governors meeting to declare that Iran was non-compliant of NPT.
Choose the most appropriate answer from the options given below:

1 A only 2 B only

3 C only 4 A and B only


EXPLANATION
▪ The following points are made by Ashley J. Tellis, in the context of India-US relations on how since 2001
India, despite its formal commitment to nonalignment, has supported the United States in many areas :

▪ India offered Unqualified support (100% complete support) for the US war in Afghanistan, including an offer
of use of numerous Indian military bases. Hence, Statement A is Wrong

▪ India remained silent on the US President President Bush’s decision to withdrawal from the ABM Treaty.
Hence, Statement B is also Wrong

▪ India vote with the US, and against Iran at the September 2005 IAEA Board of Governors meeting –
to declare that Iran is being ‘non-compliant’ with the Non-Proliferation Treaty. Hence, Statement C is Correct

▪ These are examples where India since 2001 has entered into a larger and long-term strategic relationship
with the US on convergent issues of strategic interests.
▪ Therefore, Statements A and B are not included, while C is included as per the Question NTA has asked.
SOURCES

Source : Challenge and Strategy: Rethinking India′s Foreign Policy by Rajiv Sikri. Pg. 189,190.
CURRENT
AFFAIRS
QUESTION
INDIA’S FOREIGN POLICY

6. Which among the following countries is not a member of NAM?

1 Burkina Faso 2 Angola

3 Cameroon 4 South Sudan


CURRENT
AFFAIRS
QUESTION
INDIA’S FOREIGN POLICY

6. Which among the following countries is not a member of NAM?

1 Burkina Faso 2 Angola

3 Cameroon 4 South Sudan


EXPLANATION

▪ Currently, there are a total 120 NAM Member Countries.

▪ Currently every African country (except South Sudan) is a


member of the Non Aligned Movement.

▪ There are 53 Countries from - Africa, 39 - from Asia,


26 - from Latin America & Caribbean, and 2 - from Europe.
SOURCES

Source : Non-Aligned Movement. Wikipedia. Source : Members and other Participants of NAM
Link : https://en.wikipedia.org/wiki/Non-Aligned_Movement Movement. Ministry of External Affairs.
Link : https://www.mea.gov.in/Images/pdf/Members-
and-other-participants.pdf
DIRECT
FIRST TIME
QUOTES
INDIA’S FOREIGN POLICY

7. Who among the following Prime Ministers spoke about India’s approach
towards the US and Soviet Union drifting towards a Cold War as
“They are neighbours in Asia and inevitably we shall have to undertake many
common tasks and have much to do with each other.”

1 Jawaharlal Nehru 2 P. V. Narsimha Rao

3 Atal Bihari Vajpayee 4 Manmohan Singh


DIRECT
FIRST TIME
QUOTES
INDIA’S FOREIGN POLICY

7. Who among the following Prime Ministers spoke about India’s approach
towards the US and Soviet Union drifting towards a Cold War as
“They are neighbours in Asia and inevitably we shall have to undertake many
common tasks and have much to do with each other.”

1 Jawaharlal Nehru 2 P. V. Narsimha Rao

3 Atal Bihari Vajpayee 4 Manmohan Singh


EXPLANATION
▪ In April 1945, amidst the pre-independence period, the Soviet Foreign Minister,
while addressing the San Francisco Conference of the United Nations - showing
support for India's quest for independence, stated, "We have at this Conference
an Indian delegation, but India is not an independent State. We all know that
the time will come when the voice of an Independent India will be heard, too."

▪ Following the Second World War, Soviet leaders and media frequently discussed
India's political situation, openly endorsing its demand for independence.

▪ Jawaharlal Nehru conveyed India's regards to the Soviet Union in an All India
Radio broadcast on September 7, 1946, said – "They are our neighbours in Asia
and inevitably we shall have to undertake many common tasks and have much
to do with each other."

▪ These events set the stage for the subsequent relations between the two
countries during the Cold War era after India gained independence.
SOURCES

Source : India-Mongolia Relations: Beyond Greater Central Asia edited by Sureyya Yigit , Dr R Srinivasan. Pg . 8 .
DIRECT
FIRST TIME
FACTS INDIA’S FOREIGN POLICY

8. Rajiv Gandhi’s participation at the Six Nation Five Continent Peace Initiative
was a campaign :-

1 To end nuclear testing.


2 For integration with Africans regional institutions.
3 For import substitution.
4 To end apartheid regime.
DIRECT
FIRST TIME
FACTS INDIA’S FOREIGN POLICY

8. Rajiv Gandhi’s participation at the Six Nation Five Continent Peace Initiative
was a campaign :-

1 To end nuclear testing.


2 For integration with Africans regional institutions.
3 For import substitution.
4 To end apartheid regime.
EXPLANATION
▪ A Six-nation disarmament and peace initiative took place in 1985 in Stockholm.

▪ PM Rajiv Gandhi attended this summit along with leaders from Argentina, Greece, Sweden, Mexico, Tanzania

▪ The Six-nation Initiative was a group formed in 1984 to lobby for Nuclear Disarmament amidst the Cold War
Arms Race.

▪ It is important to note that the Six-nation initiative, was initially started by PM Indira Gandhi in May 1984,
at a time when the dialogue between the major powers (USA & USSR) was at a standstill.
But it matured into an Action Plan only under the tenure of PM Rajiv Gandhi.

▪ PM Rajiv Gandhi said at the Stockholm Summit of the Six Nations Initiative – “the resources meant for
weaponry must be channeled for economic development and the pursuit of dominance must be replaced by
co-existence and cooperation.”
SOURCES

Source : A-Z of India's Foreign Policy Ebook (Eduseeker) . Pg . 30 .


HOW TO PREPARE FOR DECEMBER EXAM 2023 ?

▪ To FOCUS ON –
▪ Read on Major Ongoing Themes in India’s Foreign Policy
▪ Eg :- Indian Ocean Region, Strategic Autonomy,
PM Modi’s Foreign Policy, Dr. Jaishankar’s contributions

UNIT – 6
▪ To FOCUS ON –
▪ Read News Paper Articles/Editorials in IFP – Related to
India’s Foreign Policy Agreements, Events, Summits.
INDIA’S
FOREIGN POLICY

▪ Reference Material :-
▪ Eduseeker’s A-Z of India’s Foreign Policy Detailed E-Book.
▪ Oxford Handbook of India’s Foreign Policy.
▪ For Current Affairs – The Hindu / Indian Express Editorials.
POLITICAL
INSTITUTIONS
OF INDIA
DIRECT
FIRST TIME POLITICAL INSTITUTIONS OF INDIA
FACTS

1. Limited Franchise was introduced by which of these Acts?

1 Indian Council Act, 1892 2 Government of India Act, 1935

3 India Independence Act, 1947 4 None of these


DIRECT
FIRST TIME POLITICAL INSTITUTIONS OF INDIA
FACTS

1. Limited Franchise was introduced by which of these Acts?

1 Indian Council Act, 1892 2 Government of India Act, 1935

3 India Independence Act, 1947 4 None of these


EXPLANATION
▪ The Indian Council Act of 1892, introduced the concept of "Limited Franchise" in the legislative councils of
British India.
▪ There was an increase in the size of various legislative councils in India thereby increasing the engagement of
Indians with respect to the administration in British India.
▪ To elect members of the councils, an indirect election system was implemented.
▪ The principle of election, which was accepted in 1892, allowed non-officials to have a free and open
discussion on the government's financial strategy. As a result, the administration had an opportunity to clear
up misconceptions and respond to criticism.
▪ The act made it clear that the members appointed to the council were not there as representatives of any
Indian body, but as nominees of the Governor-General.
▪ Indian Members could for the first time Debate the Budget, but without having the ability to vote on it.
They were also barred from asking follow-up questions.
▪ The elected members were permitted to discuss official and internal matters.
▪ Despite the fact that Indians did not have the power to veto the majority, their opinions were heard.
It was the first step toward a representative system of government in contemporary India.
SOURCES

Source : The Indian Parliament: A Democracy at Work by B.L. Shankar, Valerian Rodrigues. Pg . 39 .
DIRECT
NTA POLITICAL INSTITUTIONS OF INDIA
PYQs

2. NITI Aayog was established through the Cabinet Secretariat


resolution dated?

1 01-12-2014 2 01-01-2015

3 01-01-2016 4 01-12-2016
DIRECT
NTA POLITICAL INSTITUTIONS OF INDIA
PYQs

2. NITI Aayog was established through the Cabinet Secretariat


resolution dated?

1 01-12-2014 2 01-01-2015

3 01-01-2016 4 01-12-2016
EXPLANATION
▪ On January 1st 2015, the NITI Aayog (National Institution for Transforming India) was
established as the successor to the Planning Commission.

▪ It was created by an executive resolution of the Government of India (i.e., Union Cabinet).

▪ Hence, it is also neither a constitutional body nor a statutory body.

▪ In other words, it is a non-constitutional or extra-constitutional body (i.e., not created by


the Constitution) and a non-statutory body (not created by an Act of the Parliament).

▪ NITI Aayog is the premier policy ‘Think Tank’ of the Government of India, providing both
directional and policy inputs to the Government of India.
SOURCES

Source : Indian POLITY by M. Laxmikanth. Pg. 971 .


DIRECT
NTA
SYLLABUS POLITICAL INSTITUTIONS OF INDIA

3. Arrange the following Committees/Commissions on local governments in chronological


order:
A. G.V.K Rao Committee
B. Ashok Mehta Committee
C. Thungon Committee
D. L.M. Singhvi Committee

Choose the correct answer from the options given below:


1 A, D, C, B 2 B, A, D, C

3 A, B, C, D 4 A, C, D, B
DIRECT
NTA
SYLLABUS POLITICAL INSTITUTIONS OF INDIA

3. Arrange the following Committees/Commissions on local governments in chronological


order:
A. G.V.K Rao Committee
B. Ashok Mehta Committee
C. Thungon Committee
D. L.M. Singhvi Committee

Choose the correct answer from the options given below:


1 A, D, C, B 2 B, A, D, C

3 A, B, C, D 4 A, C, D, B
EXPLANATION

▪ Major Committees/Commissions on Local Governments in India in Chronological Order:

➢ Ashok Mehta Committee – 1977

➢ G.V.K Rao Committee – 1985

➢ L.M. Singhvi Committee – 1986

➢ P.K. Thungon Committee – 1988


SOURCES

Source : Rapid Fire Revision Ebook (Eduseeker) . Pg. 163 and 164.
DIRECT
NTA POLITICAL INSTITUTIONS OF INDIA
SYLLABUS

4. Which among the following is not included in the Right to Life under
Article 21 of the Indian Constitution as interpreted by the
Supreme Court?
1 Right to suicide 2 Right to privacy

3 Right against public hanging 4 Right to good health


DIRECT
NTA POLITICAL INSTITUTIONS OF INDIA
SYLLABUS

4. Which among the following is not included in the Right to Life under
Article 21 of the Indian Constitution as interpreted by the
Supreme Court?
1 Right to suicide 2 Right to privacy

3 Right against public hanging 4 Right to good health


EXPLANATION
▪ Article 21 declares – that no person shall be deprived of his life or personal liberty except according to
procedure established by law.
▪ The expression ‘Personal Liberty’ in Article 21 is of the widest amplitude and it covers a variety of rights that
go to constitute the personal liberties of a man.
▪ The Supreme Court has declared the following rights as part of Article 21 :-
(1) Right to live with human (8) Right to free legal aid. (15) Right against bonded (22) Right of women to be (29) Right to social and economic
dignity. labour. treated with decency and justice and empowerment.
dignity.
(2) Right to decent environment (9) Right against solitary (16) Right against custodial (23) Right against (30) Right against bar fetters.
including pollution free water and confinement. harassment. public hanging.
air and protection against
hazardous industries.
(3) Right to livelihood. (10) Right to speedy trial. (17) Right to emergency medical (24) Right to road in hilly areas. (31) Right to appropriate life
aid. insurance policy.
(4) Right to privacy. (11) Right against handcuffing. (18) Right to timely medical (25) Right to information. (32) Right to sleep.
treatment in government
hospital.
(5) Right to shelter. (12) Right against inhuman (19) Right not to be driven out of (26) Right to reputation. (33) Right to freedom from noise
treatment. a state. pollution.
(6) Right to health. (13) Right against delayed (20) Right to fair trial. (27) Right of appeal from a (34) Right to sustainable
execution. judgement of conviction. development.
(7) Right to free education up to (14) Right to travel abroad. (21) Right of prisoner to have (28) Right to family pension. (35) Right to opportunity.
14 years of age. necessities of life.
SOURCES

Source : Indian POLITY by M. Laxmikanth. Pg. 203, 204 .


DIRECT
FIRST TIME
FACTS POLITICAL INSTITUTIONS OF INDIA

5. Which of these postulated the introduction of an elected President in the


municipalities?

1 Lord Ripon’s Resolution, 1882


2 Lord Mayo’s Resolution, 1870
3 Bombay Municipal Corporation Act, 1888
4 Gujrat Municipalities Act, 1964
DIRECT
FIRST TIME
FACTS POLITICAL INSTITUTIONS OF INDIA

5. Which of these postulated the introduction of an elected President in the


municipalities?

1 Lord Ripon’s Resolution, 1882


2 Lord Mayo’s Resolution, 1870
3 Bombay Municipal Corporation Act, 1888
4 Gujrat Municipalities Act, 1964
EXPLANATION
▪ In 1870, Lord Mayo, the Viceroy of India, introduced a resolution
aiming to decentralize authority and establish local self-government,
involving more representation from Indians.

▪ The resolution sought to develop self-governance, enhance municipal


institutions, and involve Indians and Europeans more in
administration.

▪ Until then, local governance initiatives were driven by revenue or


basic urban needs, with British-dominated, nominated bodies.

▪ Lord Mayo’s 1870 Resolution – proposed the introduction of an


Elected President in the Municipalities.
SOURCES

Source : INDIAN POLITICS: CONSTITUTIONAL FOUNDATIONS AND INSTITUTIONAL FUNCTIONING by M.P. SINGH, REKHA SAXENA. Pg. 243.
DIRECT
NTA
SYLLABUS POLITICAL INSTITUTIONS OF INDIA

6. Which of the following statements is/are correct regarding parliamentary proceedings?


A. Starred questions requires an oral answer and supplementary questions can follow.
B. Short notice question is asked by giving a notice of less than 2 days.
C. The Adjournment motion needs the support of 50 members to admit it.
D. The Zero hour starts immediately after the question hour and lasts until the agenda for
the day.

Choose the correct answer from the options given below:


1 A, B, C and D only 2 A, C and D only

3 B and D only 4 A, B and C only


DIRECT
NTA
SYLLABUS POLITICAL INSTITUTIONS OF INDIA

6. Which of the following statements is/are correct regarding parliamentary proceedings?


A. Starred questions requires an oral answer and supplementary questions can follow.
B. Short notice question is asked by giving a notice of less than 2 days.
C. The Adjournment motion needs the support of 50 members to admit it.
D. The Zero hour starts immediately after the question hour and lasts until the agenda for
the day.

Choose the correct answer from the options given below:


1 A, B, C and D only 2 A, C and D only

3 B and D only 4 A, B and C only


EXPLANATION

▪ Statements A, C, D are correct statements with regards to Parliamentary Proceedings :-


▪ Statement A - A starred question is distinguished by an asterisk mark * and it requires an oral answer and
hence supplementary questions can follow.
▪ Statement C - Adjournment Motion is introduced in the Parliament to draw attention of the House to a
definite matter of urgent public importance, and needs the support of 50 members to be admitted.
▪ Statement D - The Zero hour starts immediately after the question hour and lasts until the agenda for the day
(ie, regular business of the House) is taken up. In other words, the time gap between the question hour and
the agenda is known as Zero hour. It is an Indian innovation in the field of parliamentary procedures and has
been in existence since 1962.

▪ While, Statement B - Short notice question is asked by giving a notice of less than 2 days. – is WRONG.
▪ It should be less than 10 days, not 2 days. A Short notice question is answered orally.
SOURCES

Source : Indian POLITY by M. Laxmikanth. Pg. 508, 510 .


DIRECT
NTA
SYLLABUS POLITICAL INSTITUTIONS OF INDIA

7. Which of the following statements is/are correct regarding Supreme Court


of India?
A. Under the Arbitration and Conciliation Act 1996, International Commercial
Arbitration can also be initiated in the Supreme Court.
B. The Supreme Court has special advisory jurisdiction under Article 143.
C. Article 32 of the Constitution gives an extensive original jurisdiction to the
Supreme Court with regard to enforcement of Fundamental Rights.
Choose the correct answer from the options given below:

1 A and B only 2 B and C only

3 A and C only 4 A, B, C only


DIRECT
NTA
SYLLABUS POLITICAL INSTITUTIONS OF INDIA

7. Which of the following statements is/are correct regarding Supreme Court


of India?
A. Under the Arbitration and Conciliation Act 1996, International Commercial
Arbitration can also be initiated in the Supreme Court.
B. The Supreme Court has special advisory jurisdiction under Article 143.
C. Article 32 of the Constitution gives an extensive original jurisdiction to the
Supreme Court with regard to enforcement of Fundamental Rights.
Choose the correct answer from the options given below:

1 A and B only 2 B and C only

3 A and C only 4 A, B, C only


EXPLANATION

▪ All the three Statements are True with regards to Jurisdiction of the Supreme Court of India :-

▪ A. Under the Arbitration and Conciliation Act 1996, International Commercial Arbitration can also
be initiated in the Supreme Court.

▪ B. The Supreme Court has special advisory jurisdiction under Article 143.

▪ C. Article 32 of the Constitution gives an extensive original jurisdiction to the Supreme Court with
regard to enforcement of Fundamental Rights.
SOURCES

Source : Supreme Court of India. Jurisdiction.


Link : https://main.sci.gov.in/jurisdiction
DIRECT
NTA
SYLLABUS POLITICAL INSTITUTIONS OF INDIA

8. Which of the following statements is/are correct regarding Central Information


Commission (CIC) ?
A. CIC was constituted through an official gazette notification under the
provisions of the RTI Act 2005.
B. When CIC was constituted initially, it has five commissions including Chief
Information Commissioners.
C. The Chief Information Commissioner holds office for a period of three years
from the date on which he enters upon office.
Choose the correct answer from the options given below:
1 A and B only 2 A and C only

3 B and C only 4 A, B and C only


DIRECT
NTA
SYLLABUS POLITICAL INSTITUTIONS OF INDIA

8. Which of the following statements is/are correct regarding Central Information


Commission (CIC) ?
A. CIC was constituted through an official gazette notification under the
provisions of the RTI Act 2005.
B. When CIC was constituted initially, it has five commissions including Chief
Information Commissioners.
C. The Chief Information Commissioner holds office for a period of three years
from the date on which he enters upon office.
Choose the correct answer from the options given below:
1 A and B only 2 A and C only

3 B and C only 4 A, B and C only


EXPLANATION

▪ All the three Statements are True with regards to Central Information Commission (CIC) :-

▪ A. CIC was constituted through an official gazette notification under the provisions of the
RTI Act 2005.

▪ B. When CIC was constituted initially, it has five commissioners including Chief Information
Commissioners.

▪ C. The Chief Information Commissioner holds office for a period of three years from the
date on which he enters upon office.
SOURCES

Source : Indian POLITY by Laxmikanth. Pg . 1013,1014 .

Source : Central Information Commission. Link : https://cic.gov.in/who-are-we


HOW TO PREPARE FOR DECEMBER EXAM 2023 ?

▪ To FOCUS ON –
▪ Read Not Only Articles/Amendments but ALSO FOCUS ON
The Sub-points within these Articles.
▪ Prepare in-depth points for POLITY.

UNIT – 7
▪ To FOCUS ON –
▪ Read ALWAYS in Chronological Order – Committees,
POLITICAL Articles, Amendments, Landmark Judgements.
INSTITUTIONS
OF INDIA
▪ Reference Material :-
▪ Eduseeker’s Rapid Fire Revision Ebook.
▪ Indian POLITY by M. Laxmikanth.
POLITICAL
PROCESSES
IN INDIA
DIRECT
FIRST TIME
QUOTES
POLITICAL PROCESSES IN INDIA

1. Who among the following argued that, “when NGOs are tied up
with social movements, they have occasionally changed their political
agendas”?
1 Neera Chandhoke 2 Gurpreet Mahajan

3 Niraja Gopal Jayal 4 Manoranjan Mohanty


DIRECT
FIRST TIME
QUOTES
POLITICAL PROCESSES IN INDIA

1. Who among the following argued that, “when NGOs are tied up
with social movements, they have occasionally changed their political
agendas”?
1 Neera Chandhoke 2 Gurpreet Mahajan

3 Niraja Gopal Jayal 4 Manoranjan Mohanty


EXPLANATION

▪ The context of this question is how collaboration between NGOs and grassroots social movements can lead
to changes in the political agendas of the NGOs.
▪ When NGOs work together with local groups fighting for important causes, their plans for what they want to
achieve politically can change.
▪ Neera Chandhoke explains that when NGOs and these groups team up closely, the NGOs can adjust their
goals to match what the groups are trying to achieve.
▪ For instance, there’s an example she gives about ARCH-Vahini, an NGO that which focuses on areas of rural
health and development in Gujarat.
▪ At first, members of ARCH-Vahini supported the goals of the Narmada Bachao Andolan (NBA) movement
that aimed to protect people from the dam project.
▪ But as time passed, ARCH-Vahini began to doubt the way the NBA leaders were doing things.
▪ This shift in their perspective led to accusations of having an ARCH-Vahini "NGO mindset", that is,
not understanding the local people's feelings.
▪ Therefore, this shows us that when NGOs get involved with these groups, their ideas about what they should
do politically can change based on how the group is working.
SOURCES

Source : Oxford Companion to Politics in India . Pg . 431 .


DIRECT
FIRST TIME
NEW THINKER POLITICAL PROCESSES IN INDIA

2. Which of the two factors enhanced the growth rate of Indian economy in the 1980s
as per Arvind Panagariya?
A. Attitudinal change amongst the policy makers.
B. Expert incentives for private companies. Export incentives
C. Political leadership was skeptical about import substitution industrialization
D. A more realistic exchange rate policy.

Choose the correct answer from the options given below:


1 B and D only 2 B and C only

3 D and C only 4 A and B only


DIRECT
FIRST TIME
NEW THINKER POLITICAL PROCESSES IN INDIA

2. Which of the two factors enhanced the growth rate of Indian economy in the 1980s
as per Arvind Panagariya?
A. Attitudinal change amongst the policy makers.
B. Expert incentives for private companies. Export incentives
C. Political leadership was skeptical about import substitution industrialization
D. A more realistic exchange rate policy.

Choose the correct answer from the options given below:


1 B and D only 2 B and C only

3 D and C only 4 A and B only


EXPLANATION
▪ Arvind Panagariya highlights two primary factors that drove the growth during the 1980s :
▪ Firstly, the concept of Liberalization was significant. This involved making it easier for the country to trade with other
nations by reducing import restrictions, providing incentives to boost exports, and adjusting the exchange rate in a more
practical manner. This approach improved how efficiently the economy worked.
▪ Additionally, within the country, the process of removing limitations on investments in various industries complemented
the international trade liberalization and sped up the growth of the industrial sector.
▪ Secondly, the Government's ability to Borrow Money, both from foreign sources and domestically,
was crucial. This money was then used for substantial government spending, which in turn created
higher demand for goods and services and thus played a role in driving economic growth.

▪ The point regarding ‘Attitudinal change’ was put forth by Rodrik and Subramanian who criticized and challenged the
viewpoint of Arvind Panagariya.
▪ While they both agreed that there were some actions promoting 'liberalization' during the 1980s,
their argument is that the shift in policy during that time was more about changing the government's attitude towards
businesses and the private sector. It wasn't primarily driven by concrete policy changes aimed at making the economy
more liberal through either opening up to international trade or removing internal restrictions. These changes were
designed to help existing businesses make more money without putting them at risk of facing strong competition.
SOURCES

Source : Oxford Companion to Politics in India . Pg . 445 .


DIRECT
NTA
SYLLABUS POLITICAL PROCESSES IN INDIA

3. Arrange the following political parties in a chronological order according to their


year of formation :
A. Samajwadi Party
B. Bharatiya Janata Party
C. CPI (M)
D. Shiv Sena

Choose the correct answer from the options given below:


1 B, C, D, A 2 D, B, A, C

3 C, D, B, A 4 C, A, B, D
DIRECT
NTA
SYLLABUS POLITICAL PROCESSES IN INDIA

3. Arrange the following political parties in a chronological order according to their


year of formation :
A. Samajwadi Party
B. Bharatiya Janata Party
C. CPI (M)
D. Shiv Sena

Choose the correct answer from the options given below:


1 B, C, D, A 2 D, B, A, C

3 C, D, B, A 4 C, A, B, D
EXPLANATION

▪ Political Parties in a chronological order according to their Year of Formation :

➢ CPI (M) – 1964

➢ Shiv Sena – 1966

➢ Bharatiya Janata Party – 1980

➢ Samajwadi Party – 1992


SOURCES

Source : Rapid Fire Revision ebook (Eduseeker) . Pg . 170,172,173 .


SOURCES

Source : TOP 100 JRF Booster MCQs (Eduseeker) . Q . 76 .


DIRECT
FIRST TIME
CONCEPTS POLITICAL PROCESSES IN INDIA

4. Which of the following statements does NOT indicate deepening of


democracy facilitated by gendering politics in India?

1 Women participating in politics as a community or group.


2 Articulation of political voice of women.
3 Women from upper caste carrying out a silent revolution.
4 Women getting involved in the public sphere.
DIRECT
FIRST TIME
CONCEPTS POLITICAL PROCESSES IN INDIA

4. Which of the following statements does NOT indicate deepening of


democracy facilitated by gendering politics in India?

1 Women participating in politics as a community or group.


2 Articulation of political voice of women.
3 Women from upper caste carrying out a silent revolution.
4 Women getting involved in the public sphere.
EXPLANATION
▪ In the context of India, the concept of democratization doesn't translate into increased participation of individuals, as
often discussed in liberal ideas. This is also evident in how women participate in politics.
▪ They don't participate as individual women, but as representatives of specific community identities.
Community-identity seems to be the governing force.
▪ Women are getting involved in the public sphere not as atomized individuals, as individual women, but they are getting
involved as representatives of specific community identities.
▪ These community-based identities have a strong influence. Even though marginalized groups, often referred to as
peripheral groups, pursue similar political objectives, they maintain their connections to social groups like caste, religion,
or sect as they engage in politics. This consistent move toward democratization has given voice to voices that were
previously neglected, leading to the articulation of a political voice of women.
▪ By allowing marginalized groups to express themselves, Indian democracy has weakened traditional social limitations and
empowered these groups to believe in their capacity to improve their social and economic situations. This process marks
a silent revolution, a substantial transfer of power from upper caste elites to various subaltern groups within the
framework of democratic governance.
▪ The statement "Women from upper caste carrying out a silent revolution." is wrong.
▪ Here, the ‘Silent Revolution’ marks the meaningful transfer of power - from upper caste elites to various disadvantaged
subgroups who are now getting back the agency to articulate their political voice – by themselves.
SOURCES

Source : Indian Government and Politics by Bidyut Chakrabarty. Pg . 314 .


DIRECT
FIRST TIME
QUOTES
POLITICAL PROCESSES IN INDIA

5. Who among the following scholars gave the statement that,


“In course of time, however (while Nehru was still alive), the Congress party
ceased to be a movement and became a party and government”?

1 James Manor 2 Morris Jones

3 Rajni Kothari 4 Yogendra Yadav


DIRECT
FIRST TIME
QUOTES
POLITICAL PROCESSES IN INDIA

5. Who among the following scholars gave the statement that,


“In course of time, however (while Nehru was still alive), the Congress party
ceased to be a movement and became a party and government”?

1 James Manor 2 Morris Jones

3 Rajni Kothari 4 Yogendra Yadav


EXPLANATION
▪ In the given quote, Rajni Kothari points out the distinction between the Congress party's dominance during
Nehru's era and its dominance under Indira Gandhi.

▪ Rajni Kothari mentions that Indian National Congress (INC) party, started as a movement that fought for
India's independence, symbolizing a broader national struggle. However, as time went on, especially while
Nehru was still alive, the Congress party shifted from being primarily a movement with a broad focus to
becoming more focused on being a political party and a governing entity engaged in government affairs.

▪ Rajni Kothari further highlights that under Indira Gandhi's leadership, the transformation continued.
The Congress party's identity as a broad-based political party also gradually faded, and it became
increasingly synonymous with being in power as the government.

▪ Later, during the Emergency period (1975-1977), even the government's mechanisms seemed to dissolve,
leaving behind a more centralized and exclusive decision-making group, referred to as a "caucus.“

▪ Kothari calls this transformation – “The Indian party system appears to have moved full circle."
SOURCES

Source : Indian Government and Politics by Bidyut Chakrabarty. Pg . 216 .


DIRECT
NTA
SYLLABUS POLITICAL PROCESSES IN INDIA

6. Match List I with List II :


List-I I(Books) List-II (Thinkers)
A. Politics and Ethics of the Indian I. B. L. Shankar and Valerian Rodrigues
Constitution
B. The Indian Parliament II. K. C. Wheare
C. Federal Government III. Rajiv Bhargava
D. Indian Constitution: Cornerstone of a IV. Granville Austin
Nation
1 A-III, B-II, C-I, D-IV 2 A-III, B-I, C-II, D-IV
3 A-II, B-I, C-III, D-IV 4 A-IV, B-III, C-II, D-I
DIRECT
NTA
SYLLABUS POLITICAL PROCESSES IN INDIA

6. Match List I with List II :


List-I I(Books) List-II (Thinkers)
A. Politics and Ethics of the Indian I. B. L. Shankar and Valerian Rodrigues
Constitution
B. The Indian Parliament II. K. C. Wheare
C. Federal Government III. Rajiv Bhargava
D. Indian Constitution: Cornerstone of a IV. Granville Austin
Nation
1 A-III, B-II, C-I, D-IV 2 A-III, B-I, C-II, D-IV
3 A-II, B-I, C-III, D-IV 4 A-IV, B-III, C-II, D-I
EXPLANATION

▪ The Given Books and their Authors matched correctly :-


➢ Politics and Ethics of the Indian Constitution – Rajiv Bhargava
➢ The Indian Parliament – B.L. Shankar and Valerian Rodrigues
➢ Federal Government – K. C. Wheare
➢ Indian Constitution: A Cornerstone of a Nation – Granville Austin
DIRECT
FIRST TIME
CONCEPTS POLITICAL PROCESSES IN INDIA

7. Which of the following statements have not been made by M.N. Srinivas regarding
Caste Politics in India?
have been made
A. There is horizontal solidarity of caste in India.
B. Modern technology created hurdles on the way to caste solidarity.
C. There was even vertical solidarity between castes.
D. There was competition amongst castes in political economic sphere.

Choose the correct answer from the options given below:


1 A and B only 2 B and C only

3 B, C and D only 4 A, C and D only


DIRECT
FIRST TIME
CONCEPTS POLITICAL PROCESSES IN INDIA

7. Which of the following statements have not been made by M.N. Srinivas regarding
Caste Politics in India?
have been made
A. There is horizontal solidarity of caste in India.
B. Modern technology created hurdles on the way to caste solidarity.
C. There was even vertical solidarity between castes.
D. There was competition amongst castes in political economic sphere.

Choose the correct answer from the options given below:


1 A and B only 2 B and C only

3 B, C and D only 4 A, C and D only


EXPLANATION
▪ These statements were made by M.N. Srinivas on the dual impact of Modern technology and Representational Politics on
Caste Politics in India :
▪ A. “There is horizontal solidarity of caste in India.” – Horizontal solidarity in the caste system refers to the social cohesion
and support that exists within a particular caste group.
▪ C. “There was even vertical solidarity between castes.” – Vertical solidarity refers to the hierarchical nature of the caste
system. It involves the relationship between different caste groups and is characterized by unequal power dynamics.
▪ D. “There was competition amongst castes in political economic sphere.” – At the village level castes were
interdependent and aware of it. There was a system of annual grain-payments made to the Smith, Potter, Barber,
Washerman and Priest in the village.
▪ While each caste had its own solidarity, it was also aware of its solidarity with other castes—each Smith, for instance,
competed with all other Smiths for the custom of the landowners. Besides this competition, other kinds of ties such as
those between master and servant, landlord and tenant, creditor and debtor, and patron and client cut across the
divisions of caste. Therefore, there was existing competition amongst castes in the political economic sphere of the
village.
▪ Statement B – “Modern technology created hurdles on the way to caste solidarity” is False – because –
▪ Modern technological advancements like printing, communication, education, and political awareness spurred caste
groups to unite, discuss shared concerns, and resulting in a greater increase in caste solidarity.
▪ Hence, Modern technology did not hinder the rise, instead, it helped to facilitate the rise of caste solidarity.
EXPLANATION
▪ In Conclusion, M.N. Srinivas mentions that with the introduction of modern technology and rapid improvement in
communications during British India - castes found it possible to range over wide areas and as a result the horizontal
solidarity of a caste gained at the expense of the vertical solidarity of the castes of a region.

▪ The British administration's implementation of certain forms of representative politics played a significant role in
promoting the horizontal consolidation of caste groups. This was achieved through policies that granted power to local
self-governing bodies and offered preferences and concessions to the backward castes.
▪ These policies created new avenues for castes to enhance their influence. To capitalize on these opportunities, traditional
caste groups formed alliances, working together to create larger entities.

▪ This process, however, was not a one-sided affair. Parallelly, the caste system itself was undergoing changes.
▪ The increased horizontal solidarity among castes, driven by their competition for political and economic advantages,
gradually weakened the vertical solidarity of caste. In the past, this vertical solidarity meant that higher castes exercised
authority over lower ones.
▪ This transformation was further accelerated with the introduction of democratic politics after India gained independence.
The shift toward democratic governance and political representation further influenced the dynamics of caste
interactions, marking a significant change in the traditional system.
SOURCES

Source : Oxford Companion to Politics in India . Pg . 158 .


DIRECT
FIRST TIME
BOOK
POLITICAL PROCESSES IN INDIA

8. Who among the following scholars has written the book


“Remapping India: New states and their political origins”?

1 Ashutosh Varshney 2 Louisie Tillin

3 Paul Brass 4 Suhas Palshikar


DIRECT
FIRST TIME
BOOK
POLITICAL PROCESSES IN INDIA

8. Who among the following scholars has written the book


“Remapping India: New states and their political origins”?

1 Ashutosh Varshney 2 Louisie Tillin

3 Paul Brass 4 Suhas Palshikar


EXPLANATION

▪ The book "Remapping India : New States and Their Political Origins" by Louise Tillin
draws attention to two significant points :

▪ Firstly, it takes a look at the role of new social movements in contributing to the
decline of the Indian National Congress Party's dominance, particularly the concept
of 'one-party dominance,' starting from the early 1970s.
▪ Secondly, it highlights the emergence of the Bharatiya Janata Party (BJP) as a
contender to the Congress's political supremacy, which became more pronounced
after 1980.
▪ Remapping India looks at the most recent episode of state creation in 2000, when
the states of Chhattisgarh, Jharkhand and Uttarakhand came into being.
How their creation represented a new turn in the history of the country’s territorial
organisation.
▪ This book explains the politics that lay behind this episode of ‘post-linguistic’ state
re-organisation and what it means for the future design of India’s federal system.
DIRECT
NTA
SYLLABUS POLITICAL PROCESSES IN INDIA

9. Match List I with List II :


List-I (States/Union Territories) List-II (Year of Establishment)
A. Punjab I. 1966
B. Gujarat II. 1960
C. Ladakh III. 2014
D. Telangana IV. 2019

1 A-I, B-II, C-IV, D-III 2 A-I, B-II, C-III, D-IV


3 A-II, B-I, C-IV, D-III 4 A-II, B-I, C-III, D-IV
DIRECT
NTA
SYLLABUS POLITICAL PROCESSES IN INDIA

9. Match List I with List II :


List-I (States/Union Territories) List-II (Year of Establishment)
A. Punjab I. 1966
B. Gujarat II. 1960
C. Ladakh III. 2014
D. Telangana IV. 2019

1 A-I, B-II, C-IV, D-III 2 A-I, B-II, C-III, D-IV


3 A-II, B-I, C-IV, D-III 4 A-II, B-I, C-III, D-IV
EXPLANATION
▪ Gujarat – On May 1st 1960, the state of ▪ Telangana – With the Andhra Pradesh
Gujarat was formed with the state of Reorganisation Act of 2014,
Bombay divided into two states i.e. Telangana became India’s 29th state
Maharashtra and Gujarat by the Bombay on June 2nd 2014.
(Reorganisation) Act 1960.

▪ Punjab – With The Punjab Reorganisation ▪ Ladakh – On 5th August 2019 with the
Act, 1966 - Three States Punjab, Haryana Jammu and Kashmir Reorganisation
and Himachal Pradesh were established in Act Jammu & Kashmir became a UT
1966. with legislature and Ladakh became a
UT without legislature.

TOPIC :- RE-ORGANISATION OF STATES OF INDIA


SOURCES

Source : Rapid Fire Revision ebook (Eduseeker) . Pg . 181 .


HOW TO PREPARE FOR DECEMBER EXAM 2023 ?
▪ To FOCUS ON –
▪ Focus Equally on Conceptual Chapters + Factual Part for Unit-8
▪ Read Not Only the Most Popular Thinkers on State Politics in
India. Focus on a Wider Range of Thinkers like – Read Chapters
written by Palshikar, Varshney, Neera Chandhoke, Zoya
Hassan, Rob Jenkins, Aseema Sinha, Yogendra Yadav, Sudipta
Kaviraj etc.

UNIT – 8 ▪ To FOCUS ON –
▪ Also Do Prepare Social Movements, Political Parties,
Reorganization of States in Chronological Order.
POLITICAL PROCESSES ▪ ALWAYS REMEMBER – To Prepare Factual Part in
IN INDIA Chronological Order format. Questions are framed like this.

▪ Reference Material :- (Cover In-depth from Any One Textbook)


▪ Oxford Companion to Politics in India.
▪ Indian Government and Politics by Bidyut Chakrabarty.
▪ Eduseeker’s Famous Commentators of Indian Politics E-book.
PUBLIC
ADMINISTRATION
DIRECT
NTA
SYLLABUS PUBLIC ADMINISTRATION

1. Which of the following statements is not correct regarding management according to


Peter Drucker?
A. Management is subjective
B. Management has a specific role to perform
C. Management is designed for ordinary citizens too
D. Management approves the mechanistic model of organizational management

Choose the correct answer from the options given below:


1 B and D only 2 C and D only

3 A and B only 4 A and D only


DIRECT
NTA
SYLLABUS PUBLIC ADMINISTRATION

1. Which of the following statements is not correct regarding management according to


Peter Drucker?
A. Management is subjective
B. Management has a specific role to perform
C. Management is designed for ordinary citizens too
D. Management approves the mechanistic model of organizational management

Choose the correct answer from the options given below:


1 B and D only 2 C and D only

3 A and B only 4 A and D only


EXPLANATION
▪ According to Peter Drucker’s Management by Objectives (MBO) :-

▪ Statements B and C are correct.


▪ "Management has a specific role to perform" - Peter Drucker believed that management has a specific role to perform within an
organization. He defined management as the process of planning, organizing, directing, and controlling an organization's resources
to achieve its goals and objectives.
▪ "Management is designed for ordinary citizens too" - Peter Drucker advocated for the democratization of management. He
believed that management principles should not be reserved for a select group of experts or elites but should be accessible and
applicable to individuals from all walks of life.

▪ Statements A and D are incorrect.


▪ "Management is subjective" - This statement is incorrect because according to Peter Drucker, management is not subjective. In
fact, he emphasized that management is a discipline that can be studied, learned, and practiced objectively. Drucker believed that
there are principles and practices that can be applied universally to achieve effective management.
▪ "Management approves the mechanistic model of organizational management" - This statement is incorrect. Peter Drucker was
actually critical of the mechanistic model of organizational management. He believed that organizations should be more flexible,
adaptive, and people-centered. He advocated for a more organic and human-centered approach to management, which is in
contrast to the rigid and hierarchical nature of the mechanistic model.
DIRECT
NTA
SYLLABUS PUBLIC ADMINISTRATION

2. Match List I with List II :


List-I List-II
A. Rational choice Theory of Public I. Emulates private sector practices
Administration
B. New Public Administration II. Encourages research in policy analysis
C. New Public Management III. Devices principles and laws of public
administration
D. Golden Era of Public Administration IV. Focus of norms and client loyalty

1 A-I, B-III, C-IV, D-II 2 A-II, B-IV, C-I, D-III


3 A-III, B-II, C-I, D-IV 4 A-IV, B-III, C-II, D-I
DIRECT
NTA
SYLLABUS PUBLIC ADMINISTRATION

2. Match List I with List II :


List-I List-II
A. Rational choice Theory of Public I. Emulates private sector practices
Administration
B. New Public Administration II. Encourages research in policy analysis
C. New Public Management III. Devices principles and laws of public
administration
D. Golden Era of Public Administration IV. Focus of norms and client loyalty

1 A-I, B-III, C-IV, D-II 2 A-II, B-IV, C-I, D-III


3 A-III, B-II, C-I, D-IV 4 A-IV, B-III, C-II, D-I
EXPLANATION

▪ A. Rational Choice Theory of Public Administration – Encourages research in policy analysis


▪ Rational Choice Theory of Public Administration encourages research in policy analysis because it aligns
with the theory's core assumption that individuals are rational actors who carefully assess their options
to maximize their interests. Policy analysis helps decision-makers identify optimal policies by providing a
systematic framework to evaluate alternatives, anticipate consequences, and ultimately make more
informed and effective choices in the realm of public administration.

▪ B. New Public Administration – Focus of norms and client loyalty


▪ New Public Administration advocates a client-focused approach. It stresses not only on providing goods
and services to the clients but also giving them a voice in how and when and what is to be provided.
It requires positive, proactive and responsive administrators rather than authoritarian and ivory-tower
bureaucrats.
EXPLANATION

▪ C. New Public Management – Emulates private sector practices


▪ New public management has emerged out of the Thatcherism and Reaganism (USA) of the 1980s. The
emphasis of new public management is on performance-appraisal, managerial autonomy, cost-cutting,
financial incentives, output targets, innovation, responsiveness, competence, accountability, market-
orientation, quality improvement, contracting out, flexibility, competition, choice, information
technology, debureaucratisation, decentralization, down-sizing and entrepreneurialism. The new public
management involves a shift from direct provision of services by government to indirect methods like
policy-making, facilitating, contracting, providing information and coordinating other actors.
▪ D. Golden Era of Public Administration – Devices principles and laws of public administration
▪ The era during 1927-1937 is called the "Golden Era" of Public Administration. During this stage, the
scholars believed that there are certain principles of administration which could be discovered and
applied to increase the efficiency and economy of public administration. They argued that
administration is administration irrespective of the nature and context of work because the principles of
administration have universal validity and relevancy. Hence, they claimed that public administration is a
science. Public administration reached its reputational zenith during this stage. Thinkers belonging to
this era are – W.F. Willoughby, M.P. Follet, Henri Fayol, Mooney and Reiley, Gulick and Urwick.
SOURCES

Source : Public Administration by M. Laxmikanth. Pg . 38,41,13,14.


DIRECT
NTA
PYQs PUBLIC ADMINISTRATION

3. Arrange the following events/conferences of public administration in chronological


order:
A. The Philadelphia Conference
B. First Minnowbrook Conference
C. Publication of book “Public Administration in a time of Turbulence” by Waldo
D. Publication of “The Elements of Administration” by Urwick

Choose the correct answer from the options given below:


1 B, A, C, D 2 D, B, C, A

3 A, D, C, B 4 D, A, B, C
DIRECT
NTA
PYQs PUBLIC ADMINISTRATION

3. Arrange the following events/conferences of public administration in chronological


order:
A. The Philadelphia Conference
B. First Minnowbrook Conference
C. Publication of book “Public Administration in a time of Turbulence” by Waldo
D. Publication of “The Elements of Administration” by Urwick

Choose the correct answer from the options given below:


1 B, A, C, D 2 D, B, C, A

3 A, D, C, B 4 D, A, B, C
EXPLANATION

▪ Major events/conferences of Public Administration in chronological order: :

➢ Publication of “The Elements of Administration” by Urwick – 1944

➢ The Philadelphia Conference – 1967

➢ First Minnowbrook Conference – 1968

➢ Publication of book “Public Administration in a time of Turbulence”


by Waldo - 1971
SOURCES

Source : Public Administration by M. Laxmikanth. Pg . 36,37,45,46.


DIRECT
NTA
SYLLABUS PUBLIC ADMINISTRATION

4. According to MP Follett, which of the following is not a principle of


organisation in achieving coordination?

1 Coordination as the reciprocal relation of all factors in a situation.


2 Coordination by indirect control.
3 Coordination in early stages.
4 Coordination as a continuing process.
DIRECT
NTA
SYLLABUS PUBLIC ADMINISTRATION

4. According to MP Follett, which of the following is not a principle of


organisation in achieving coordination?

1 Coordination as the reciprocal relation of all factors in a situation.


2 Coordination by indirect control.
3 Coordination in early stages.
4 Coordination as a continuing process.
EXPLANATION
▪ According to MP Follet, there are 4 principles of achieving Coordination in an organisation –

▪ 1. Coordination by Direct Contact: Coordination is best achieved through direct


communication and collaboration among individuals or departments, ensuring mutual
understanding and resolving issues promptly.

▪ 2. Coordination in the Early Stages: Effective coordination begins at the outset of any
project, preventing conflicts and ensuring a smooth workflow as tasks progress.

▪ 3. Coordination as a Continuing Process: Coordination should be a continuous process, not


a one-time event. Coordination is an ongoing effort that adapts to changing circumstances
and evolving needs, sustaining efficient operations throughout the project's duration.

▪ 4. Coordination as the Reciprocal Phenomenon: Coordination involves a reciprocal


relationship where various parts of an organization cooperate and adjust their actions to
align with one another, fostering unity and synergy.
SOURCES

Source : Rapid Fire Revision ebook (Eduseeker) . Pg . 184 .


DIRECT
NTA
PYQs PUBLIC ADMINISTRATION

5. Who amongst these are meant to serve the shop floor according to Fredrick Taylor’s
functional foremanship in his scientific management theory?
A. Speed Boss
B. Repair Boss
C. Route clerk
D. Time clerk
E. Inspection foreman
Choose the correct answer from the options given below:
1 C, D & E only 2 B, D & A only

3 A, B & E only 4 B, A & C only


DIRECT
NTA
PYQs PUBLIC ADMINISTRATION

5. Who amongst these are meant to serve the shop floor according to Fredrick Taylor’s
functional foremanship in his scientific management theory?
A. Speed Boss
B. Repair Boss
C. Route clerk
D. Time clerk
E. Inspection foreman
Choose the correct answer from the options given below:
1 C, D & E only 2 B, D & A only

3 A, B & E only 4 B, A & C only


EXPLANATION

▪ Functional Foremanship is a management system developed by F.W. Taylor in which the planning and
execution of work is divided among eight specialized supervisors.
▪ Of the Eight Functional Foremen – Four are responsible for Planning and sit in the Planning Room.

Order-of-Work Instruction-Card Time-and-Cost Shop


and Route Clerk Clerk Clerk Disciplinarian

▪ The other Four Functional Foremen are responsible for Execution and serve on the Shop Floor.

Gang boss Speed boss Inspector Repair boss

▪ The goal of functional foremanship is to improve Efficiency and Productivity by having each worker receive
specialized instruction and supervision from the experts.
SOURCES

Source : Public Administration by M. Laxmikanth. Pg . 105 .


DIRECT
FIRST TIME
FACTS
PUBLIC ADMINISTRATION

6. Which period of the evolution of public administration is referred as


‘The Exploratory Period’?

1 1887-1910 2 1910-1947

3 1947-1970 4 1970-1980
DIRECT
FIRST TIME
FACTS
PUBLIC ADMINISTRATION

6. Which period of the evolution of public administration is referred as


‘The Exploratory Period’?

1 1887-1910 2 1910-1947

3 1947-1970 4 1970-1980
EXPLANATION

▪ The 1880s were the seedtime the origins of the study of Public Administration.
▪ In the Exploratory Period (1887 – 1910) the primary focus of this period was on Reform
(managing the public service at minimal cost), administrative neutrality, politics-administration dichotomy
and introduction of certain courses related to public administration in certain universities.

▪ The major thrust of this period was on extension of administrative practice – which was believed to be a
necessary step in improving a government.

▪ Some Major works of this Exploratory Period (1887 – 1910) :-


▪ The publication of 1887 essay ‘The Study of Administration’ by Woodrow Wilson discussed that public
administration could be studied outside of politics. His intent was to remove administration from the paws of
politicians.
▪ Goodnow's ‘Politics and Administration’ (1900) is generally regarded as the first true textbook in public
administration where he explored the nature of politics-administration dichotomy.
SOURCES

Source : Administrative Theories and Management Thought by R.K. Sapru . Pg. 14 .


DIRECT
NTA
SYLLABUS PUBLIC ADMINISTRATION

7. Assertion (A): A prismatic society has heterogenous structures existing side


by side and which are diametrically opposite to each other.

Reason (R): The social change in a prismatic society would be inconsistent,


incomplete and unresponsive.

1 Both A and R are true and R is the correct explanation of A


2 Both A and R are true but R is NOT the correct explanation of A
3 A is correct but Statement R is false
4 A is not correct but Statement R is true
DIRECT
NTA
SYLLABUS PUBLIC ADMINISTRATION

7. Assertion (A): A prismatic society has heterogenous structures existing side


by side and which are diametrically opposite to each other.

Reason (R): The social change in a prismatic society would be inconsistent,


incomplete and unresponsive.

1 Both A and R are true and R is the correct explanation of A


2 Both A and R are true but R is NOT the correct explanation of A
3 A is correct but Statement R is false
4 A is not correct but Statement R is true
EXPLANATION

▪ When discussing a Prismatic Society, “Heterogeneity" refers to the coexistence of diverse structures.
▪ This means that within such a society, various elements with contrasting characteristics exist side by side.
This diversity extends to viewpoints and practices, even those that are diametrically opposite.
▪ The consequence of such parallel existence is that any societal change occurring within this prism of diversity
tends to be inconsistent, lacking in completeness, and unresponsive to uniform transformation.
▪ Riggs points out that one might observe the coexistence of highly modernized structures alongside deeply
traditional ones.
▪ For instance, Urban areas, influenced by Western patterns and trends, undergo a different trajectory
compared to rural regions, where traditional ways of life persist. This coexistence results in an uneven
developmental landscape.
▪ All these complexities gives rise to challenges and intricacies within the society's fabric where the level of
social change taking place within a prismatic society would be inconsistent, incomplete and unresponsive
from place to place.

▪ Hence, Both A and R are true and R is the correct explanation of A.


SOURCES

Source : Administrative Theories and Management Thought by R.K. Sapru . Pg. 278 .
DIRECT
NTA
SYLLABUS
PUBLIC ADMINISTRATION

8. Who among the following has written the books ‘Public Policy
Reexamined’ and ‘Design for Policy Sciences’?

1 Harold Lasswell 2 Thomas R Dye

3 Rosolio Peter 4 Yehezkel Dror


DIRECT
NTA
SYLLABUS
PUBLIC ADMINISTRATION

8. Who among the following has written the books ‘Public Policy
Reexamined’ and ‘Design for Policy Sciences’?

1 Harold Lasswell 2 Thomas R Dye

3 Rosolio Peter 4 Yehezkel Dror


EXPLANATION
▪ Yehezkel Dror has written the books ‘Public Policy Re-examined’ (1968) and ‘Design for Policy Sciences’ (1971).

▪ Yehezkel Dror in his Book, "Public Policy Re-examined" (1968) has talked of the policy making process
in the Developing and Developed countries.

▪ According to him, in Developing States, policy making is based on inheritance and legacy. Policy strategy is not consciously determined
in developing States. There is lack of systematic institutional arrangements for learning feedback. In some of the cases, just trial and
error methods are applied.
▪ Voters have rather insignificant role to play in policy making. Role of intellectuals is also limited. Legislatures have a secondary role to
play in policy making as political executive is stronger. There is lack of proper knowledge and information with those who are part and
parcel of the process. The optimal excellence of policy making is much low as policy making is of an average quality.
▪ Policy making process in Developed States is realistic, having economic and political feasibility. Prioritisation of values and operational
goals are well-spelt out. Separate institutions exist in some of the countries for the purpose of undertaking long-range policy-making
surveying; knowledge for handling research and development of policy making.
▪ In democratic systems, the voters exert important power on policy makers through elections. The intellectuals play a major and
substantive role in policy making. The Heads of the Governments have prominent and most significant role to play in policy making.
Legislature as an institution has a major role to play in policy-making in democratic systems and a minimal role in dictatorial systems.
Policy making in developed States is up to the mark for ensuring their survival and satisfying the genuine needs and wants of the
population.
EXPLANATION
▪ Yehezkel Dror in his Book, ‘Design for Policy Sciences’ (1971). has listed nine elements of Policy Sciences.
▪ These 9 elements are intended to guide and structure the process of developing effective policies.
Here's a breakdown of each element:
▪ 1. Clarification of Values, Objectives, and Decision Criteria: Clearly define the values, objectives, and criteria
that will guide the decision-making process. This ensures that the policy aligns with the desired outcomes.
▪ 2. Identification of Alternatives: Identify various alternatives for addressing the issue at hand. Encourage the exploration of
new alternatives by surveying existing literature, experiences, and theories.
▪ 3. Preliminary Estimation of Expected Payoffs: Estimate the potential outcomes or benefits associated with each alternative.
▪ 4. Selection of Decision Model: Depending on the nature of the issue and the expected payoffs, determine whether to follow an incremental-
change model (focusing on gradual improvements) or a more innovative approach.
▪ 5. Establishment of Cutoff Horizon: For issues involving minimal risk or innovation, define a cutoff horizon for evaluating the potential results of
alternative policies. Identify major expected outcomes using available knowledge and intuition.
▪ 6. Quantitative and Qualitative Analysis: Analyze alternatives considering both quantitative (economic) and qualitative (political) factors. This
approach aims to overcome the limitations of current systems analysis and advance toward policy analysis.
▪ 7. Significance Assessment: Evaluate whether the issue is significant enough to warrant more comprehensive analysis. Blend theory and
experience, based on their availability and relevance to the problem.
▪ 8. Use of Explicit Techniques: Employ explicit techniques such as simulation and the Delphi method when appropriate. To draw knowledge from
various disciplines to enhance the analysis and decision-making process.
▪ 9. Arrangements for Systematic Learning and Improvement: Incorporate mechanisms for learning from experience, fostering initiative and
creativity, developing the skills of staff members, and promoting intellectual efforts to continually enhance the quality of policy-making.
SOURCES

Source : Public Administration by M. Laxmikanth. Pg . 46 .


DIRECT
NTA
PYQs
PUBLIC ADMINISTRATION

9. Who among the following thinker proposal the concept of


“Mental Revolution”?

1 F. W. Taylor 2 Elton Mayo

3 Chester Barnard 4 H. Simon


DIRECT
NTA
PYQs
PUBLIC ADMINISTRATION

9. Who among the following thinker proposal the concept of


“Mental Revolution”?

1 F. W. Taylor 2 Elton Mayo

3 Chester Barnard 4 H. Simon


EXPLANATION
▪ The concept of ‘Mental Revolution’ by F. W. Taylor refers to a change in the attitude of both
management and workers towards each other.

▪ He believed that for scientific management to be successful, there must be a fundamental change in
the way that both parties think about their roles and responsibilities.

▪ Management must stop viewing workers as lazy and inefficient, and start seeing them as partners in
the production process. They must also be willing to invest in training and development, so that
workers can be more productive.

▪ Workers must stop viewing management as their adversary, and start seeing them as allies in the
quest for efficiency. They must also be willing to put in the effort to learn new skills and work more
efficiently.

▪ Thus ‘Mental Revolution' (or the ‘new outlook’ according to Taylor) requires the realisation on part
of both the parties (workers and managers) that their mutual interest is not contradictory and both
can prosper only through cooperation, not conflict. This is the essence of Scientific Management.
SOURCES

Source : Public Administration by M.Laxmikanth.


Pg . 103 .

Source : Crash Course on Public Administration


(Eduseeker) . Pg. 6 .
SOURCES

Source : All India JRF-NET Political Science FREE MOCK TEST (Eduseeker) . Q . 56 .
DIRECT
FIRST TIME
QUOTES
PUBLIC ADMINISTRATION

10. Who among the following thinkers said that, “The communication
of intangible facts, opinions, suggestions and suspicions that cannot
pass through formal channels are communicated through informal
channels”?
1 Hebert Simon 2 Max Weber

3 Chris Argryis 4 Chester Barnard


DIRECT
FIRST TIME
QUOTES
PUBLIC ADMINISTRATION

10. Who among the following thinkers said that, “The communication
of intangible facts, opinions, suggestions and suspicions that cannot
pass through formal channels are communicated through informal
channels”?
1 Hebert Simon 2 Max Weber

3 Chris Argryis 4 Chester Barnard


EXPLANATION

▪ The following quote is said by Chester I. Barnard in the context of Formal and Informal Organizations.

▪ Chester Barnard introduced the term ‘Informal Organization' to the management literature.
He describes an Informal Organization as a ‘system of unconscious activities and is essentially structureless.’

▪ According to Barnard, there are 3 positive functions which can be performed ONLY by informal organizations:
▪ 1. To perform "the communication of intangible facts, opinions, suggestions, suspicions that cannot pass
through formal channels without raising issues"
▪ 2. To maintain cohesiveness in formal organizations
▪ 3. To help maintain the feeling of personal integrity, of self-respect, of independent choice. It may be
regarded as "a means of maintaining the personality of individual against certain effects of formal
organizations which tend to disintegrate the personality“

▪ It is the first function of Informal Orgs which was quoted in the Question asked by NTA.
SOURCES

Source : Administrative Theories and Management Thought by R.K. Sapru . Pg. 194 .
HOW TO PREPARE FOR DECEMBER EXAM 2023 ?

▪ To FOCUS ON –
▪ Just Do Not Read Factual Parts. This time Complete
Along with Detailed Points given by the Main PubAd Thinkers.
▪ Focus on Classical Thinkers of Pub Ad (Fayol, Taylor, Simon)

UNIT – 9
▪ To FOCUS ON –
▪ Focus on the Features part , Bullet Points given in
Textbooks. The Questions will come from in-depth points.
PUBLIC
ADMINISTRATION

▪ Reference Material :- (Cover In-depth from Any One Textbook)


▪ Eduseeker’s Crash Course on Public Administration.
▪ Public Administration by M. Laxmikanth.
▪ Administrative Theories & Management Thought by R.K.Sapru.
GOVERNANCE
AND PUBLIC POLICY
IN INDIA
DIRECT
FIRST TIME
FACTS
GOVERNANCE AND PUBLIC POLICY IN INDIA

1. Which of the following websites is a comprehensive website of Citizen


Charters of Government of India?

1 www.goecharters.nic.in
2 www.goindiacharters.nic.in
3 www.goicharters.nic.in
4 www.echartersind.nic.in
DIRECT
FIRST TIME
FACTS
GOVERNANCE AND PUBLIC POLICY IN INDIA

1. Which of the following websites is a comprehensive website of Citizen


Charters of Government of India?

1 www.goecharters.nic.in
2 www.goindiacharters.nic.in
3 www.goicharters.nic.in
4 www.echartersind.nic.in
EXPLANATION

▪ On 31st May 2002, a comprehensive Website of Citizens' Charter in Government of India


(www.goicharters.nic.in) was developed and launched by the
Department of Administrative Reforms and Public Grievances (DARPG).

Source : www.goicharters.nic.in
SOURCES

Source : DARPG. Citizens Charters – Historical Background. Link : https://shorturl.at/brK69


DIRECT
NTA
SYLLABUS
GOVERNANCE AND PUBLIC POLICY IN INDIA

2. Place the following legislative acts in chronological orders:


A. Right to Information Act
B. Consumer Protection Act Consumer Protection Amendment Act
C. Right to Education Act
D. Lokpal and Lokayukta Act
Choose the correct answer from the options given below:

1 B, A, D, C 2 A, C, D, B

3 A, B, C, D 4 C, A, D, B
DIRECT
NTA
SYLLABUS
GOVERNANCE AND PUBLIC POLICY IN INDIA

2. Place the following legislative acts in chronological orders:


A. Right to Information Act
B. Consumer Protection Act Consumer Protection Amendment Act
C. Right to Education Act
D. Lokpal and Lokayukta Act
Choose the correct answer from the options given below:

1 B, A, D, C 2 A, C, D, B

3 A, B, C, D 4 C, A, D, B
EXPLANATION

▪ Major Legislative Acts in chronological order are as follows :

➢ Right to Information Act – 2005

➢ Right to Education Act – 2009

➢ Lokpal and Lokayukta Act – 2013

➢ Consumer Protection (Amendment) Act – 2019


EXPLANATION

▪ A. Right to Information Act – 2005. It was introduced by Second Administrative Reforms


Commission which has submitted a total of 15 reports and ‘Right to Information: Master Key to
Good Governance’ was among them.

▪ C. Right to Education Act – 2009. This act describes importance of free and compulsory education
for children between 6-14 years in India under Article 21(a) of the Indian Constitution.

▪ D. Lokpal and Lokayukta Act – 2013. This act established the institution of Lokpal at the Centre and
the Lokayukta at the State level.

▪ B. Consumer Protection (Amendment) Act – 2019. An amendment was brought on the Consumer
Protection Act, 1986 on August 9th, 2019. It widened the definition of consumer and recognize
mediation as an alternative disputes resolution mechanism.
SOURCES

Source : Rapid Fire Revision ebook (Eduseeker) . Pg . 193,194,195,197.


DIRECT
FIRST TIME
FACTS
GOVERNANCE AND PUBLIC POLICY IN INDIA

3. Which of the following Laws/Rules are restricting the Disclosure of


Official Information to the people?
A. Official Secrets Act, 1923
B. Commission of Enquiry Act, 1952
C. Indian Evidence Act, 1812 1872
D. Fifth Pay Commission
Choose the correct answer from the options given below:
1 A, B, C, D only 2 A, B and C only

3 B, C and D only 4 A, B and D only


DIRECT
FIRST TIME
FACTS
GOVERNANCE AND PUBLIC POLICY IN INDIA

3. Which of the following Laws/Rules are restricting the Disclosure of


Official Information to the people?
A. Official Secrets Act, 1923
B. Commission of Enquiry Act, 1952
C. Indian Evidence Act, 1812 1872
D. Fifth Pay Commission
Choose the correct answer from the options given below:
1 A, B, C, D only 2 A, B and C only

3 B, C and D only 4 A, B and D only


EXPLANATION

▪ In India, there are various laws and rules which restrict the disclosure of official
information to the people and thus favour secrecy in administration.

▪ These laws and rules are as follows :


▪ Official Secrets Act, 1923
▪ Indian Evidence Act, 1872
▪ Commission of Enquiry Act, 1952
▪ All-India Services (Conduct) Rules, 1954
▪ Central Civil Services (Conduct) Rules, 1955
▪ Railway Services (Conduct) Rules, 1956
SOURCES

Source : Governance in India by M. Laxmikanth. Pg . 43 .


DIRECT
NTA
PYQs
GOVERNANCE AND PUBLIC POLICY IN INDIA

4. Which of the following Commissions/Committees recommended for the


establishment of the Lokpal and Lokayukta for First time?

1 First Administrative Reforms Commission


2 Santhanam Committee
3 A. D. Gorwala Committee
4 IInd Pay Commission
DIRECT
NTA
PYQs
GOVERNANCE AND PUBLIC POLICY IN INDIA

4. Which of the following Commissions/Committees recommended for the


establishment of the Lokpal and Lokayukta for First time?

1 First Administrative Reforms Commission


2 Santhanam Committee
3 A. D. Gorwala Committee
4 IInd Pay Commission
EXPLANATION

▪ The creation of Lokpal at the Centre and Lokayukta at the State level was recommended by the
First Administrative Reforms (ARC) commission in the year 1966.

▪ Then, the first Lokpal Bill was introduced in Parliament in 1968.

▪ After ten official attempts, from 1968 to 2011 the Government of India finally passed the Lokpal and
Lokayuktas Bill, of 2011 was passed in the year 2013.

▪ In 2011, the famous Anna Movement for Lokpal took place at Jantar Mantar, Delhi.

▪ The Lokpal and Lokayuktas Act, 2013 came into force w.e.f. 16.01.2014.

▪ This established the institution of Lokpal at the Centre and the Lokayukta at the State level.
SOURCES

Source : Indian POLITY by Laxmikanth. Pg . 1053.

Source : Rapid Fire Revision ebook (Eduseeker) . Pg. 195 .


DIRECT
NTA GOVERNANCE AND PUBLIC POLICY IN INDIA
PYQs

5. In which year did the amendment to the Constitution extend the


Panchayati Raj provisions to the scheduled areas?

1 1992 2 1995

3 1996 4 1997
DIRECT
NTA GOVERNANCE AND PUBLIC POLICY IN INDIA
PYQs

5. In which year did the amendment to the Constitution extend the


Panchayati Raj provisions to the scheduled areas?

1 1992 2 1995

3 1996 4 1997
EXPLANATION
▪ The 73rd Constitutional Amendment, 1993 excluded the adivasi and the scheduled areas from its purview.

▪ But it was always open for the Parliament to extend to same to these areas, as per the provisions of the Article 243M(4).

▪ Bhuria Committee was constituted in 1994 to formulate law for extending the provisions of the Part IX to the Scheduled
areas and to suggest modifications in other acts relevant to the Fifth (V) schedule in order to further strengthen the local
self-governing areas.

▪ The recommendation of this committee culminated in the enactment of the Panchayats (Extension to the Scheduled
Areas) (PESA) Act 1996, which came into effect on 24th December 1996.

▪ The PESA Act has led to the extension of the panchayats act to the tribal areas of Andhra Pradesh, Orissa, Gujarat,
Chhattisgarh, Himachal Pradesh, Jharkhand, Maharashtra, Madhya Pradesh, and Rajasthan.

▪ This enables the tribal society to effectively take control of their traditional rights over natural resources.
SOURCES

Source : Governance in India by M. Karthikeyan. Pg . 63 .


HOW TO PREPARE FOR DECEMBER EXAM 2023 ?

▪ To FOCUS ON –
▪ Cover Major Government of India Schemes.
▪ Background, Dates, Features, Recommendations of Schemes.
▪ Focus on Dates of Laws Passed, Enactment, Applied.

UNIT – 10
▪ To FOCUS ON –
▪ Focus on the Features part , Bullet Points given in
GOVERNANCE AND Textbooks. The Questions will come from in-depth points.
PUBLIC POLICY
IN INDIA
▪ Reference Material :- (Cover In-depth from Any One Textbook)
▪ Eduseeker’s Rapid Fire Revision Ebook.
▪ Governance in India by M. Karthikeyan.
▪ Governance in India by M. Laxmikanth.
Dear J.R.F. ASPIRANTS,
YOU CAN PURCHASE OUR E-books, Mock Paper Test Series, Resources :-

Get COMBO Package Deal :-


PAPER 2
POLITICAL SCIENCE
ALL 10 Units - Rapid Fire Revision Ebook
Unit-wise , Topic-wise Revision Ebook
(With Keywords of Thinkers)
(With Simple Quick Revision Format Charts)
+
Last 5 Years ALL 10 Units PYQs Solved Booklet
with Detail Explanation of Each PYQ Question.

Website Link to Ebooks =>


https://eduseeker.in/products/bulk-
2/63efaa2dd514e36db0015e16
Dear J.R.F. ASPIRANTS,
YOU CAN PURCHASE OUR E-books, Mock Paper Test Series, Resources :-

RAPID FIRE REVISION NOW IN HINDI :-


पेपर 2
राजिीनत नवज्ञाि
सभी 10 यूनिट का रै नपड फायर ररवीजि ईबुक
यूनिट के अिुसार, नविय के अिुसार पूणय रूप से
ररवीजि ईबुक

+
नपछिे 5 विों NTA के सभी 10 यूनिट के प्रश्न पनिका
पूणयतः हि नकताब
प्रत्येक प्रश्न की नवस्तृत व्याख्या के साथ।

Website Link to Ebooks =>


https://eduseeker.in/products/bulk-
2/6476f9600f899c5c10fab0d1
Dear J.R.F. ASPIRANTS,
YOU CAN PURCHASE OUR E-books, Mock Paper Test Series, Resources :-

Get COMBO Package Deal :-


Unit – 6
A-Z of India’s Foreign Policy
Prime Minister-wise All Major Events, Timeline of
India’s Foreign Policy (1947 – 2023 covered)

+
Last 10 Years of India’s Foreign Policy PYQs
Questions Solved with Detailed Analysis.

+
Last 6 Months Current Affairs
I.R. & India’s Foreign Policy with Practice MCQs.
Website Link to Ebooks =>
https://eduseeker.in/products/bulk-
2/63e1d38f717ccc06248b2bad
Dear J.R.F. ASPIRANTS,
YOU CAN PURCHASE OUR E-books, Mock Paper Test Series, Resources :-

PAPER 2
Unit – 3
Indian Political Thought E-book
(Most Detailed Method All IPT Thinkers Covered
with Detailed Explanation of Each Concept, Books
and Authors, Timeline of Events, Video Links)

Link => https://eduseeker.in/products/pdf-


4/601fcffd4bed3e35821e800a

Unit – 5
International Relations E-book
(All Theories Covered, All Organisations Covered
With Easy Bullet Points Format Coverage
of International Relations Unit + Get Video Links)
Link => https://eduseeker.in/products/pdf-
4/60fd0bc7188caf3c1af4dab6
Dear J.R.F. ASPIRANTS,
YOU CAN PURCHASE OUR E-books, Mock Paper Test Series, Resources :-

PAPER 1
- Mock Paper Test Series PRACTICE -
MCQs Mastery in Just Days: NET for Paper-1
All 10 Units Paper 1
5 Full-length Mock Paper Practice (500 MCQs)
With Timer, Solved Explanation, All India Rankings
Link => https://eduseeker.in/products/test-
3/63c98f6c0253f56d3fefcafe

PAPER 2
- Mock Paper Test Series PRACTICE -
All India Political Science 2023 Test Series : Paper 2
All 10 Units Paper 2
10 Full-length Mock Paper Practice (1000 MCQs)
With Timer, Solved Explanation, All India Rankings
Link => https://eduseeker.in/products/test-
3/6119f63c8ddadc05158ddb3c
THANK
YOU

You might also like